[Ответить в тред] Ответить в тред

Check this out!


<<
Назад | Вниз | Каталог | Обновить тред | Автообновление
556 | 64 | 196

Тред тупых вопросов №91 TSIOLKOVSKY EDITION Аноним 04/06/18 Пнд 20:22:32  404988  
циолковский.jpg (80Кб, 720x409)
rocket equation.jpg (9Кб, 602x227)
TUcR7thMvWch6Mh.mp4 (3109Кб, 720x720, 00:00:25)
janus rhea satu[...].webm (284Кб, 970x492, 00:00:11)
Тред вопросов о жизни, Вселенной и всем таком.

Спрашиваем то, за что в других местах выдают путёвку в биореактор. Здесь анонимные ученые мирового уровня критически рассмотрят любые гениальные идеи и нарисованные в Paint схемы.

Предыдущий тут: >>401314 (OP)
https://2ch.hk/spc/res/401314.html

Q: Можно быстрее?
A: Можно упасть в пузырь альбукерке, наса уже почти надула его.

Q: Я начитался охуительных историй от СМИ, че делать, нам жопа?
A: Да, тебе жопа, можешь сгонять в зогач, порашу или ньюсач или куда оттуда пошлют. К историям от СМИ относись с опаской.

Q: Что будет с человеком в вакууме без скафандра / если он упадет на черную дыру / попробует ступить на поверхность газового гиганта/солнца?
A: Он умрёт.

Q: Почему бы не привязать ракету к воздушному шару или стартовать с горы?
A: Космос - это не как высоко, а как быстро, большая часть энергии ракеты уходит на разгон вбок.
Подробнее тут https://what-if.xkcd.com/58/ (английский) https://chtoes.li/orbital-speed/ (перевод)
Аноним 04/06/18 Пнд 21:56:49  405048
>>404988 (OP)
Мне с 3-ьей крипотно чет, че делать? Открываю нулевую, а оно на меня таращится, пздц
Аноним 04/06/18 Пнд 22:11:15  405066
>>405048
Это всего лишь слияние двух небольших шариков в пару километров диаметром,
В нем нету ничего злого, но оно
пугает. Но после просмотра вы
почувствуете радость, и в
дальнейшем всё в вашей жизни
будет хорошо!

Ваш друг.
Аноним 05/06/18 Втр 00:27:37  405092
Могут ли быть современные представления о чд, нейтронных звёздах, квазарах и подобном ошибочными?
Аноним 05/06/18 Втр 00:39:06  405096
>>405092
Могут.
Аноним 05/06/18 Втр 02:03:47  405102
>>405066
Анон говорит что на превью похоже на два глаза. От самой гифки только эпилептики просрутся.
Аноним 05/06/18 Втр 02:05:56  405103
image.png (121Кб, 464x494)
>>405102
Ну глаза и глаза, что ж такого-то.
Аноним 05/06/18 Втр 07:33:47  405113
>>405096
Ничего себе, честный ответ на пейсаче. Не ожидал такого. Жму руку.
Аноним 05/06/18 Втр 08:10:49  405115
>>405113
Вся квантовая механика была ошибкой. Декаденты 20х годов 20 века не могли придумать чего-то стоящего.
Аноним 05/06/18 Втр 09:23:56  405117
badumtss.png (413Кб, 783x776)
>>405115
Маня, плес.
Аноним 05/06/18 Втр 09:40:15  405119
>>405117
По существу есть возразить?
Аноним 05/06/18 Втр 11:00:58  405122
Что происходит со второй ступенью ракеты после отделения полезной нагрузки? Например у Falcon 9
Аноним 05/06/18 Втр 11:02:46  405123
>>405122
Обычно остаётся болтаться в том же месте, фалкон по возможности направляет ступень назад на землю или куда нибудь подальше.
Аноним 05/06/18 Втр 11:24:54  405129
>>405123
>фалкон по возможности направляет
Направляет или нет? Этих вторых ступеней должно там болтаться уже тысячи. Они ж сами не упадут
Аноним 05/06/18 Втр 11:27:13  405130
>>405129
Направляет если остаётся топливо после выведения.
А в целом да, там прилично ступеней болтается.
Но некоторые сами падают.
Аноним 05/06/18 Втр 12:12:04  405138
>>405122
полазь любопытства ради
http://stuffin.space/
Аноним 05/06/18 Втр 12:29:50  405141
почему на мкс нет НИГЕРОВ?
Аноним 05/06/18 Втр 12:39:22  405143
>>405141
>почему на мкс нет НИГЕРОВ?
В википедии есть 400 ссылок на американских астронавтов и где-то 20 астронавтов-негров.
То есть американец на МКС может быть ниггером с шансом около 5% и сейчас не свезло.
Аноним 05/06/18 Втр 12:48:04  405145
>>405143
>сейчас
Думаю наса поэксперементировала и успокоилась. Одно дело нигеров насильно пихать в фильмы на роль бога/тора/иисуса/гарри поттера и другое пускать гамадрила на станцию стоимостью сотню миллионов тысяч баксов
Аноним 05/06/18 Втр 12:51:50  405146
>>405145
В рамках стыковок шаттлов они точно бывали на МКС и даже выполняли там какие-то работы, но были ли они частью длительных экипажей, которые на МКС оставались - это мне каопать уже влом.
Аноним 05/06/18 Втр 13:36:21  405154
Против углеродного шовинизма есть серьезные аргументы?
Аноним 05/06/18 Втр 14:43:06  405169
>>405154
Спроси у шовинистов в /ро, хотя этот >>405145
может ответить
Аноним 05/06/18 Втр 14:43:11  405170
>>405119
Как можно возразить по существу на беспруфный вскукарек, а, зелень?

Единственное содержательное утверждение у тебя:
>Вся квантовая механика была ошибкой
очевидно ошибочно само по себе. На минуточку, предсказания той же квантовой электродинамики подтверждаются в экспериментах с огромной точностью:
https://en.wikipedia.org/wiki/Precision_tests_of_QED
Аноним 05/06/18 Втр 16:32:43  405305
1.PNG (3Кб, 216x105)
>>405138
Спасибо за залипалово
мимоанон

Заодно хочу поинтересоваться, что это за шрапнель такая?
http://stuffin.space/?search=2017-042
http://stuffin.space/?search=2018-004
http://stuffin.space/?search=2018-015

Гугл нашел информацию про первый пуск, что-то про >70 спутников за раз и всё такое, но читать стены английского текста лень. Может, кто объяснит вкратце суть, а то это больше похоже на разбрасывание мусора, чем на научные исследования.

И последнее. Что такое TBA (пикрил)?
Тут https://acronyms.thefreedictionary.com/TBA ничего подходящего не нашел.
Аноним 05/06/18 Втр 16:38:20  405314
>>405170
>подтверждаются в экспериментах
жидовские выдумки ненужны
(Автор этого поста был предупрежден.)
Аноним 05/06/18 Втр 17:07:33  405382
Спейсач, реквестирую годные книги по небесной механике
(KSP не предлагать)
Аноним 05/06/18 Втр 17:10:26  405387
>>405382
Левантовского уже прочитал?
Аноним 05/06/18 Втр 17:14:56  405394
>>405305
Я привык, что TBA=to be announced.
Возможно, создатели надеются уточнить тип объекта и со временем обновить эту информацию.
Аноним 05/06/18 Втр 17:19:42  405400
>>405387
А с чего лучше у него начать? Пытался одну прочитать, но показалась слишком сложной
Аноним 05/06/18 Втр 17:21:41  405404
>>405400
Элементарное изложение сложно?
Ну тогда может реально лучше кербач?
Аноним 05/06/18 Втр 17:26:34  405407
>>405404
эх, ладно, буду пытаться осилять
Аноним 05/06/18 Втр 17:38:52  405411
>>405305
2017-042
>14 июля 2017 года в 09:36 мск с площадки 31 космодрома БАЙКОНУР запланирован пуск ракеты-носителя «Союз-2.1а» с космической головной частью в составе разгонного блока «Фрегат», космического аппарата (КА) «Канопус-В-ИК» и 72-х малых КА попутной нагрузки. Разгонный блок «Фрегат» обеспечит выведение КА «Канопус-В-ИК» и попутных малых спутников на три различные орбиты с последующим сведением с орбиты разгонного блока. Миссия такой высокой сложности будет реализована впервые в истории запуска малых космических аппаратов.

Скандальный рекордный пуск же, там кучу кубсатов положили, до сих пор терки идут, кто виноват.
https://www.roscosmos.ru/23686/

2018-004
>India's Polar Satellite Launch Vehicle, in its forty second flight (PSLV-C40), successfully launched the 710 kg Cartosat-2 Series Satellite for earth observation and 30 co-passenger satellites together weighing about 613 kg at lift-off.
https://www.isro.gov.in/launcher/pslv-c40-cartosat-2-series-satellite-mission

>2018-015
>China launched a Long March 2D rocket from the Jiuquan Satellite Launch Center on Friday, carrying a group of seven satellites from China, Italy, Argentina and Denmark including an innovative experiment to study the ionospheric precursors of Earthquakes to evaluate whether forecasting of strong Earthquakes will be possible from an orbital vantage point.
http://spaceflight101.com/long-march-2d-launches-zhangheng-1/

Юзай разные тематические ресурсы, там по этим индексам NSSDC ID https://ru.wikipedia.org/wiki/NSSDC_ID можно найти любую информацию.
Аноним 05/06/18 Втр 17:49:46  405413
>>405394
>>405411
> Скандальный рекордный пуск же
Не следил за космосом в то время, да и сейчас не особо, прошло мимо ¯\_(ツ)_/¯

Спасибо!
Аноним 05/06/18 Втр 17:51:39  405414
>>405413
>>405305

>>333039
Аноним 05/06/18 Втр 17:56:15  405416
>>405414
Ебануться, чесслово.
Аноним 05/06/18 Втр 21:54:12  405469
14093018956190s.jpg (8Кб, 166x200)
>>405314
>Автор этого поста был предупрежден
А почему, собственно, не батут? Аноним 05/06/18 Втр 22:31:04  405473
Вот Елон Муск сажает свои ракеты на землю. Но для этого используются тяжелые опоры, которые, вместе с топливом для посадки и рулями, снижают дельту ракеты больше чем в два раза. Так почему не избавиться от тяжелых опор и не сажать ракету на батут? Почему никто не догадался? Куда подать заявку на патент?
Аноним 05/06/18 Втр 22:35:55  405476
>>405473
>снижают дельту ракеты больше чем в два раза
Откуда инфа?
>Так почему не избавиться от тяжелых опор и не сажать ракету на батут? Почему никто не догадался? Куда подать заявку на патент?
Потому, что не в два раза.
Ты еще предложи струнами ловить по бокам чтоб не накренялась.
Аноним 05/06/18 Втр 22:45:37  405483
>>405476
Сейчас не найду статью на гиктаймсе, но вроде как примерно в два раза, если сажать на платформу в океане, если сажать на землю, то еще больше. Это косвенно подтверждается тем, что ракета по массе почти как "Протон", а грузы таскает уровня "Союза", что-то тяжелее уже только в одноразовом варианте.
Аноним 05/06/18 Втр 22:47:21  405485
>>405483
Цифр у меня нет, так что спорить не возьмусь, но батут это реально омск.
Аноним 05/06/18 Втр 22:53:55  405494
загруженное.jpg (8Кб, 225x225)
f9mass.PNG (3Кб, 386x52)
>>405473
Из чего ты предлагаешь сделать огнеупорный, выдерживающий падающие с неба ~30т, диаметром более трёх метров батут?

Ладно, сделал. Как будешь стабилизировать упавшую на него ракету? Земля/баржа жёсткая, в отличие от батута.
Аноним 05/06/18 Втр 22:57:49  405496
>>405494
Так тормозить двигателями как при обычной посадке, просто оттормозившись падать на батут. И никаких опор не нужно.
Аноним 05/06/18 Втр 23:02:21  405499
om3.jpg (32Кб, 1200x800)
boom 2.jpg (4Кб, 480x360)
boom.jpg (5Кб, 268x188)
>>405496
На вопросы-то ты не ответил.

Вот упала ракета на батут, прыг-прыг и она уже летит с него в землю, дальше пикрил. Как фиксить будем?
Аноним 05/06/18 Втр 23:06:59  405501
>>405496
>>405499
Ещё одно покормлю.
Предлагаешь избавиться от опор, а чем их заменить? Поверхность натянутого батута не сильно мягче земли будет, так что сила столкновения примерно равна. Садиться на сопла двигателей а-ля семейка Керманов - так себе затея, если хочешь потом использовать их ещё раз.

Полагаю, Муск и Ко. всё же не случайно сделали
> тяжёлые опоры
, а не маленькие палочки и батут.
Аноним 05/06/18 Втр 23:14:05  405504
>>405501
Отправить Стивена ловить ракеты, очевидно же. Батут делаем в виде воронки.
Аноним 05/06/18 Втр 23:26:27  405513
Cnu2Cf-USW8.jpg (354Кб, 2405x1497)
Доставьте фото жилого отсека Союзов первых серий (не М и не Т).

Хочу посмотреть, правда ли там была деревянная столешница.
Аноним 05/06/18 Втр 23:40:34  405520
interiormuseum1.jpg (73Кб, 640x427)
>>405513
http://www.russianspaceweb.com/soyuz_bo.html
Слева вроде она.
Аноним 05/06/18 Втр 23:55:34  405527
20-1.jpg (117Кб, 1200x748)
>>405520

Не она, это уже Союз-Т или позднее.
06/06/18 Срд 00:06:24  405533
>>405527
Фото из музея National Space Centre
https://spacecentre.co.uk/blog-post/soyuz-spacecraft/
Аноним 06/06/18 Срд 00:19:16  405539
>>405520
>>405533
Там же люк наверху, АЛЛО. Это Союз-Т.

В Великобритании выставлена восстановленная копия беспилотного Союз 7К-ОК с банкой вместо Иглы. Это фото не оттуда.
Аноним 06/06/18 Срд 00:20:48  405541
>>405539
Вернее с Иглой, но в другой конфигурации
06/06/18 Срд 00:26:26  405542
>>405513
https://www.youtube.com/watch?v=Twll__UdX7k
6:55
Аноним 06/06/18 Срд 00:30:52  405543
>>405542

круто, и правда похоже! Не массив, наверное, и с рейлингами, но всё же - столешница!

Спасибо.
Аноним 06/06/18 Срд 00:34:49  405544
8-2.jpg (93Кб, 975x687)
8-1.jpg (77Кб, 1128x795)
8-3.jpg (61Кб, 784x569)
19-1.jpg (101Кб, 1200x750)
Там кстати немец какой-то книгу написал про Балашову, уже вышла. Такой-то прикладной промдизайн на острие ретрофутуризма!
Аноним 06/06/18 Срд 03:42:02  405552
262db30f35.png (42Кб, 458x145)
Известно, что у самых больших звёзд почти весь объём состоит из очень разрежённого вещества, почти вакуума. Но в то же время оно ионизировано и непрозрачно. Но на каких расстояниях эта непрозрачность проявляется? Насколько ерохин и сычёв должны отлететь друг от друга, чтобы не увидеть?
Аноним 06/06/18 Срд 10:42:57  405585
Когда фоточки с EHT подвезут?
Аноним 06/06/18 Срд 12:40:02  405606
Могут ли на внеземных объектах быть какие-то недоступные для землян ресурсы? Я краем уха что-то слышал про самородки золота на астероидах и богатые руды других металлов, насколько это правда? Может ли астероид быть с настолько высоким содержанием какого-нибудь осмия, что его теоретически было бы интересно распотрошить и отправить на Землю? Или другое вещество, которое на Земле в естественном виде не существует (или почти не существует), но представляет интерес для промышленности? Возможны условия для формирования таких астероидов?
Аноним 06/06/18 Срд 12:56:53  405609
>>405606
Фактически, всё золото, кобальт, железо, марганец, молибден, никель, осмий, палладий, платина, рений, родий и рутений, которые сейчас добываются из верхних слоёв Земли, являются остатками астероидов, упавших на Землю во время ранней метеоритной бомбардировки, когда после остывания коры на планету обрушилось огромное количество астероидного материала. Из-за большой массы более 4 млрд лет назад на Земле начала происходить дифференциация недр, в результате чего большинство тяжёлых элементов под действием гравитации опустилось к ядру планеты, поэтому кора оказалась обеднённой тяжёлыми элементами. А на большинстве астероидов из-за незначительной массы никогда не происходила дифференциация недр и все химические элементы распределены в них более равномерно.
Аноним 06/06/18 Срд 13:02:17  405611
>>405609
>все химические элементы распределены в них более равномерно.
Самородки-то будут или это всего лишь значит, что надо будет отсеять из руды не 99.9999% шлама, а целых 99.99%?
Аноним 06/06/18 Срд 13:05:40  405612
Снимок.JPG (97Кб, 752x415)
>>405611
>будут
Аноним 06/06/18 Срд 13:21:41  405614
1528280404041-1[...].jpg (624Кб, 723x1458)
>>405611
Вот эта хуйня весит 2,7*1019 кг, из них 90% железо и никель.
Аноним 06/06/18 Срд 13:50:28  405618
>>405609
Спасибо
Аноним 06/06/18 Срд 14:21:05  405632
Течёт ли кровь у самок из щелей в невесомости?
Аноним 06/06/18 Срд 14:40:53  405639
image.png (157Кб, 640x480)
>>405609
Это не совсем так. Несмотря на тяжесть материала некоторые элементы (литофильные) остались в коре. А сидерофильные (железо, кобальт, никель и платиновые с золотом) потонули в ядро, их и стоит искать в астероидах пока не откроем рилейтед.
Аноним 06/06/18 Срд 14:41:57  405642
>>405632
Естественно.
Аноним 06/06/18 Срд 15:13:23  405656
Правда что космонавты суеверные и у них есть дни(числа) в которые запуски никогда не намечаются?

Насколько я понял сейчас ракеты в космос не запускает только ленивый. Собственно вопрос зачем? и благодаря чему такая доступность технологий?

Как они не поврезались ещё в космосе?
Аноним 06/06/18 Срд 15:28:46  405663
>>405656
Насчёт дней хуй знает, но традиций дохуя.
Запускают мало кто на самом деле.
Не сталкиваются потому что там огромные расстояния.
Аноним 06/06/18 Срд 15:34:40  405666
>>405656
>космонавты суеверные
как и многие другие люди, имеющие профессии связанные с повышенными рисками

> у них есть дни(числа) в которые запуски никогда не намечаются
Я читал, что такое выжигали каленным железом еще при СССР. Суеверия - суевериями, но на работу это не должно сказываться. Числа 13 никто ну или делают вид не боится.

>сейчас ракеты в космос не запускает только ленивый
На данный момент всего полдесятка стран и столько же частников, остальные делают небольшие перделки-свистелки с пуском раз в несколько лет

>благодаря чему такая доступность технологий?
Накопленный опыт дидами, интернет, открытость технологий, развитая система привлечения инвестиций, поддержка государств. Мб контроль еще меньше чем 20-30 лет назад, палестинцы, арабы и корейцы уже давно имеют что хотели.

>Как они не поврезались ещё в космосе?
Космос, даже ближний, это много-много в степени много кубических километров ничего. Принимается ряд мер по упорядочению объектов в космосе. Например, на высоты орбиты МКС запрещено что-либо запускать чтобы исключить даже невероятно маленькую возможность столкновения. На некоторых высоких орбитах, в которых сконцентрированы спутники, международное сообщество заставляет в конце срока работы уводить аппараты на т.н. мусорные орбиты. Ну и "мусорить" поменьше. Запускать по 80 студенческих спутников там нельзя. Что летает низко, довольно быстро падает на Землю и сгорает. Все это дело контролируется с Земли, создается 3д-модель всех объектов, просчитываются безопасные траектории. Перед запуском на орбиту необходимо получить разрешение страны-хозяйки, а та в свою очередь отвечает перед ООН.
Аноним 06/06/18 Срд 15:38:41  405668
>>405642
И как они ее собирают? Там же нет гравитации, кровь вниз не потечёт.
Аноним 06/06/18 Срд 15:39:32  405670
>>405668
Сельдь, плес
Аноним 06/06/18 Срд 15:40:39  405672
>>405668
Во-первых, циклы в невесомости слегка меняются (за пруфами не полезу, может меня поправят).
Во-вторых, прокладки и тампоны по-прежнему работают.
В-третьих для истечения жидкостей из тела не нужна гравитация, они текут большей частью из-за перистальтики.
В-четвертых, вот видео как справляются на орбите с истекающими жидкостями
https://www.youtube.com/watch?v=P36xhtpw0Lg
Аноним 06/06/18 Срд 16:45:56  405680
С МКС улетают на своем союзе или предыдущем?
Аноним 06/06/18 Срд 16:50:23  405684
>>405680
На предыдущем. Неча Союзу торчать на орбите слишком долго дабы не пошло что не так.
Аноним 06/06/18 Срд 17:28:22  405688
>>404988 (OP)
Почему при образовании звезд, из газопылевого облака протопланетный диск превращается именно диск?
Почему на земле раньше было сильно теплее, хотя солнце было слабее?

Аноним 06/06/18 Срд 18:00:43  405695
>>405680
На своем, конечно, там же кресла даже индивидуально подогнаны. Полгода союз висит пристыкованный и готовый к отлету, чтобы в случае чего можно было быстро по местам рассесться и покинуть МКС.
Аноним 06/06/18 Срд 18:50:12  405697
>>405684

>>405695

Спасибо!
Аноним 06/06/18 Срд 18:50:52  405698
>>405697
Нахуй ты спасибо говоришь, не ты спрашивал.
Аноним 06/06/18 Срд 18:56:10  405699
>>405698
Мне приятно.
Аноним 06/06/18 Срд 21:02:12  405710
>>405695
Вообще-то они скручивают ложементы со своего Союза и перетаскивают в предыдущий.
Аноним 06/06/18 Срд 21:06:43  405711
>>405710
>Вообще-то они скручивают ложементы со своего Союза и перетаскивают в предыдущий.
хуяси
Аноним 06/06/18 Срд 21:19:37  405717
>>405710
Еще колеса скручивают у бесхозных припаркованных союзов и ставят их на кирпичи.
Аноним 06/06/18 Срд 22:09:27  405726
>>404988 (OP)
Скажи мне, зачем нужна МКС? Что там такого можно исследовать? Зачем держать там по полгода людей?
И почему нет чёрных на МКС?
Аноним 06/06/18 Срд 22:16:17  405727
>>405726
Платина.
Тебе - незачем. Для фундаментальной науки и исследований тема, ибо где ты еще найдешь микрограви тацию?
Аноним 06/06/18 Срд 22:18:22  405729
А правда что космонавты постоянно саботируют систему переработки мочи чтобы не пить мочу и не позориться?
Аноним 06/06/18 Срд 22:20:13  405732
>>405727
Есть падающий ИЛ-76 например.
Тогда по-другому спрошу. Нахуя она такая огромная?
Ну сделали ли бы две банки: одна спать и срать, другая работать. Для пары человек достаточно. Какого хуя они такую мразь там налепили?
Аноним 06/06/18 Срд 22:20:38  405733
>>405729
Она для технических нужд.
Аноним 06/06/18 Срд 22:21:54  405734
>>405729
Нет. Рекуперация воды из мочи идет на бытовые нужды. Пьют свежую с планеты.

>>405732
>Есть падающий ИЛ-76 например.
Может ли он падать трое суток? То-то и оно.
>Нахуя она такая огромная?
Так вышло.
>Для пары человек достаточно. Какого хуя они такую мразь там налепили?
Чтобы работало не пару человек а полдюжины
Аноним 06/06/18 Срд 22:31:09  405736
>>405734
> Чтобы работало не пару человек а полдюжины
Зачем?
Аноним 06/06/18 Срд 22:41:11  405740
>>405736
Чтоб работать, вестимо
Аноним 06/06/18 Срд 23:04:55  405749
>>405736
Станции на пару человек это пройденный этап, ниче интересного там нет.
Аноним 06/06/18 Срд 23:11:00  405751
>>405749
Скучновато, картишки не разложить, нардишки не погамать. А в компании всяко веселее.
Аноним 06/06/18 Срд 23:45:57  405754
>>405751
Да, на скайлэбе даже как то раз космонавты взбунтовались.
Аноним 06/06/18 Срд 23:50:31  405755
fukken lold8.jpg (83Кб, 682x600)
>>405754
Проиграл.
Аноним 06/06/18 Срд 23:54:26  405756
>>405755
https://2ch.hk/spc/arch/2017-12-26/res/242910.html#335231
Аноним 07/06/18 Чтв 00:00:16  405757
>>405756
Я помню этот случай, просто звучало удачно так, что я и проиграл.
Аноним 07/06/18 Чтв 01:08:43  405761
>>405656
>есть дни(числа) в которые запуски никогда не намечаются?
24 октября потому что 24 октября 1960 (говорят, что не летают, я не проверял).
Аноним 07/06/18 Чтв 01:38:54  405762
>>405761
Хз насчет пилотируемых, их не так много было, чтобы все дни гарантировано заполнить, а так летали:

два в 62, 63, 64, 67, два в 69, 74, 78, 80, 85, 89, 96, 2012

И к Марсу АМС пускали, и Восходы с Зенитами, и военные аппараты, и баллистические ракеты.
Аноним 07/06/18 Чтв 04:56:16  405765
Стикер (0Кб, 512x512)
>>405656
Я наверняка уверен, что запуски НИКОГДА не намечаются на прошлое время.
Аноним 07/06/18 Чтв 16:03:15  405845
Космонавт Людмила - правда?
Аноним 08/06/18 Птн 09:37:23  406002
Какой максимальный импульс на высококипящих топливох без скатывания в совсем ужасные вещи типа боранов?
Аноним 08/06/18 Птн 11:12:33  406008
А где срут в банках без дополнительных отсеков типа союза и лунного Аполлона?
В белом футуристчином драконе срать прямо на людях? Там хоть сортир будет или вручную говно из жопы доставать придется?
Аноним 08/06/18 Птн 11:16:45  406009
>>406008
Там место специальное за шторкой.
Аноним 08/06/18 Птн 13:07:51  406016
>>406002
https://en.wikipedia.org/wiki/Liquid_rocket_propellant
Аноним 08/06/18 Птн 15:13:45  406034
>>406008
>банках без дополнительных отсеков типа союза
У союза есть отсек, в БО толкан размещен.
> лунного Аполлона
В пакетик.
>В белом футуристчином драконе срать прямо на людях? Там хоть сортир будет
Вроде во всех новых капсулах гальюны предусмотрены.
Аноним 08/06/18 Птн 18:26:29  406053
Не могу понять, почему найдя самую дальнюю галактику на расстоянии 13 млрд лет, учёные решили, что это и есть возраст Вселенной?
Аноним 08/06/18 Птн 18:32:42  406055
>>406053
Потому, что она самая молодая?
Аноним 08/06/18 Птн 19:11:12  406062
>>406053
Это не дальняя. Наблюдаемая вселенная 42 млрд вроде, но это из-за расширения вселенной.
Аноним 08/06/18 Птн 19:13:20  406064
>>406062
Но постой! Возраст же 13 млрд лет! Откуда 42?
Аноним 08/06/18 Птн 19:14:13  406065
>>406064
Расширялась быстрее скорости света.
Аноним 08/06/18 Птн 19:35:18  406071
В крошечных галактиках на тысячу звёзд есть дыра в центре?
Аноним 08/06/18 Птн 19:50:00  406074
>>406053
>на расстоянии 13 млрд лет

Не на расстоянии 13 млрд световых лет, а свет от галактики летел к нам 13 миллиардов лет, это разные вещи. Пока фотоны летели, пространство успело нихуево так расшириться, и прямо сейчас расстояние до самых далеких галактик составляет более 30 миллиардов световых лет.

Ну и галактики не так важны, как реликтовый микроволновый фон, оставшийся с реально ранних времен (первый миллион лет после Большого взрыва), вот по нему точнее меряют.
Аноним 08/06/18 Птн 20:02:30  406081
>>405845
Нет, хуйня.
Аноним 09/06/18 Суб 01:07:29  406130
>>406065
Но быстрее света ничего нет.
Аноним 09/06/18 Суб 01:27:42  406131
>>406130
А это иной принцип. Движутся не галактики, а расширяется пространство между ними. Естественный Альбукерке короче.
Аноним 09/06/18 Суб 02:09:13  406137
Ебанный стыд...
Во-первых, Алькубьерре.
Во-вторых, не упасть, а создавать вокруг корабля изнутри (иначе кина не будет).
В-третьих, НАСА искривляет пространство на десятимиллионную часть, контролируя это сверхточными интерферометрами, до самого варп-привода здесь - как до Антарктиды раком.
Аноним 09/06/18 Суб 09:59:11  406157
>>406131
Я так понимаю, пространство внутри галактик тоже расширяется, но их не распидарашивает, потому что гравитация подсуетилась?
>>406137
Розового покемона этому хлопцу
Аноним 09/06/18 Суб 10:07:12  406158
>>406157
Вообще всё расширяется, вся вселенная. Ты тоже.
Аноним 09/06/18 Суб 10:17:20  406160
>>406157
Хуй знает почему они не рвутся, темная материя/энергия или прочая мутная хуйня.
Аноним 09/06/18 Суб 10:35:32  406164
>>406158
>Ты тоже.
Так почему я из тредов вытекаю...
Аноним 09/06/18 Суб 10:40:27  406166
>>406160
А можешь уточнить?
Аноним 09/06/18 Суб 12:14:12  406196
>>406157
>не распидорашивает
Ну возьми гандон кингсайз длиной в один километр. Поставь на нём вдоль длинной оси две отметки на расстоянии одного нанометра одна от другой. Растяни гандон в два раза. Как думаешь, на сколько увеличится расстояние между сделанными тобой ранее отметками?
Аноним 09/06/18 Суб 12:18:42  406198
>>406166
На данный момент никто не может.
Аноним 09/06/18 Суб 12:19:42  406199
>>406196
В джва раза?
Аноним 09/06/18 Суб 13:04:05  406212
>>406196
Хуевая аналогия, все не так.

Более приближенная к реальности — на гондон положили два грузика на небольшом расстоянии друг от друга, и сцепили их ниткой. Начали тянуть гондон, нитка растянулась на 0,00001%, сила упругости нитки стала больше «отрицательной силы» расширения пространства, и грузики перестали удаляться друг от друга.
Сколько бы мы гондон ни тянули, нитка не порвется и грузики не сдвинутся с места, потому что растягивающая сила зависит только от расстояния между грузиками, а оно не увеличивается.

Так же и во всей вселенной, расширение пространства не распидорашивает ни атомы, ни галактики, потому что связывающие их силы на порядки сильнее крошечного растяжения. А так как расширение пространства пропорционально только расстоянию и ничему больше, то эта растягивающая сила остается постоянной, и не распидарасит их и в будущем.

Но на действительно крупных масштабах уровня сверхскоплений галактик и выше, слабые гравитационные силы уже не могут преодолеть расширение пространства, и скопления галактик разлетаются в стороны друг от друга, причем скорость разлета, пропорциональная расстоянию, со временем все увеличивается.
Аноним 09/06/18 Суб 14:30:23  406223
>>406212
>со временем все увеличивается
Вот тут спиздел, а так все верно
Аноним 09/06/18 Суб 15:51:11  406238
Вот у нас есть 2 равноценных, 1 килограмовых куска быстро распадающегося топлива для ритега. Один кусок мы раскладываем по 100 грамм каждый, в 10 контейнеров. А второй кусок запихиваем в контейнер целиком.
Спустя н времени мы собираем 1 ритег вновь и сравниваем радиоактивность этих кусков.
Будет ли разница, в какую пользу и насколько сильно ?
Аноним 09/06/18 Суб 16:18:02  406243
>>406238
Смотря какое топливо, для стронция не будет вообще никакой разницы, а для плутония
скорее всего совсем чуточку более радиоактивны будут 100-граммовые куски, потому что осколки деления в килограммовом куске могли инициировать больше дальнейших реакций деления и чуть быстрее уработать оставшийся плутоний. Но разница там будет совсем маленькая.
Аноним 09/06/18 Суб 16:26:59  406245
>>406212
Я прост для колхозанов попытался в объяснение.
Ведь что там сраные пара светолет протяжённости галактик, если эта протяжённость изменяется в масштабах, минимум, тридцати миллиардов световых лет...
То есть, наверное, на Планковскую длину за лолиард лет. Математик-кун, скальуцлируй, плез.
Аноним 09/06/18 Суб 16:33:10  406246
1526575711725.jpg (35Кб, 221x246)
Почему бы не преобразовывать солнечную энергию в...
Ведь тогда можно было бы...
В общем, космический аппарат в космосе с практически неисчерпаемым источником энергии.
Аноним 09/06/18 Суб 16:35:35  406247
>>406246
Солнечные панели изобрел?
Аноним 09/06/18 Суб 16:36:33  406248
>>406247
Ну я имел ввиду что солнечные понели не очень сами по себе и не получили широкое распространение.
Аноним 09/06/18 Суб 16:40:04  406250
>>406248
Потому что для ловли энергии нужны огромные площади, потому что они ненадёжны, потому что чем дальше от солнца - тем хуже светит.
Аноним 09/06/18 Суб 16:41:20  406252
>>406250
Можно лампочками освещать или кострами.
Аноним 09/06/18 Суб 16:42:34  406253
>>406250
А почему нет температурных энерговыделителей? Чтобы кинул в сахару его и добывал фри энергию?
Аноним 09/06/18 Суб 16:42:50  406254
>>406248
Ну а вообще они на каждой хуйне стоят, но много от них не получишь.
Хотя даун на них до цереры долетел.
Аноним 09/06/18 Суб 16:44:35  406255
>>406253
Есть топливные элементы.
Но они кончаются, а солнце нет.
Аноним 09/06/18 Суб 17:04:47  406264
mentalnoeporno51.gif (3059Кб, 520x520)
У меня вопрос. Берём некую солнечную систему, в которой нет карланов и гигантов, но есть много землеподобных планет. Все планеты расположили свои орбиты так близко друг к другу, что у них у всех постоянно происходит орбитальная рокировка подобно взаимодействию Януса и Эпиметея у Сатурна. Только эти два меняются только друг с другом, а в данной системе ВСЕ землеподобные каменные планеты меняют орбиты от самой близкой к звезде да самой дальней.
Мне очень интересно, как может сложиться цикличность погоды, климата и даже какой может быть жизнь на одной или ан всех этих планетах.
Аноним 09/06/18 Суб 17:06:25  406265
>>406253
Низя. Либо аккумулируй тепло, римляне с зеркальным щитом, баня, пар, турбина, вот это всё, а это очень дорого, сложно и кпд херовый, либо всякие термопары, а это тоже дорого и сложно, и кпд ниже чем у фотоэлементов.
Аноним 09/06/18 Суб 17:08:26  406266
>>406265
Неужели нет никакого хайфхака на халявную энергию, ведь столько вокруг всего...
Я слыша лчто локхид мартин что-то там пилить революционное.
Аноним 09/06/18 Суб 17:13:24  406270
>>406266
Чем тебе солнечные панели не нравятся? Покрыть Сахару, вырастить боевых гмо-верблюдов для охраны периметра от мародеров, хватит на весь черный континент.

Ну хочешь, можно выращивать микрофлору, которая будет жрать #something и давать спирт на выходе, собственно в этом направлении яйцеголовые и продвигаются.
Аноним 09/06/18 Суб 17:14:17  406271
>>406270
>Ну хочешь, можно выращивать микрофлору, которая будет жрать #something и давать спирт на выходе, собственно в этом направлении яйцеголовые и продвигаются
Интересно, первый раз слышу. А на спирте что делать?
Аноним 09/06/18 Суб 17:19:31  406273
V-2kvn.jpg (53Кб, 566x690)
>>406271
Ракеты, спейсач же.
Аноним 09/06/18 Суб 20:28:25  406337
>>406253
Коассически энергия черпается из разницы потенциалов. Вот ты нагрел уде всё - и чо блчть? Где ПРОЦЕСС, при течении которого можно спиздить, взять откат на разнице?
Аноним 09/06/18 Суб 21:24:37  406359
>>406337
https://ru.wikipedia.org/wiki/Термоэлектричество
https://ru.wikipedia.org/wiki/Термоэлектрогенератор
Аноним 09/06/18 Суб 21:30:04  406362
>>406359
>совокупность явлений, в которых разница температур создаёт электрический потенциал
>разница температур

Начала термодинамики у тебя там не открыли еще, что ли? Невозможно создать полезную энергию просто из тепла, нужен еще и холодильник.
Аноним 09/06/18 Суб 21:33:53  406365
>>406362
М - Магнетизм
Аноним 09/06/18 Суб 22:38:13  406387
>>406365
Ты - хлебушек.
>>406337
>>406362
Аноним 09/06/18 Суб 23:02:14  406392
>>406212
Только ты забыл, что с увеличением расстояния сила упругости нитки увеличивается, а гравитационное притяжение ослабевает. Так что, если скорость расширения увеличивается, то рано или поздно распидарасит и голактеке и плонеты и даже атомы и даже аллаха. ПОС она такая
Аноним 09/06/18 Суб 23:05:29  406393
>>406264
Ты же понимаешь, что разница между орбитами
>Януса и Эпиметея
разница с пару сотен метров?
Аноним 09/06/18 Суб 23:27:57  406395
Солнечный зонд Паркера подлетит к Солнцу на 6 млн километров от него. От Земли до Солнца 150 млн. Значит ли это, что он будет получать в (150/6)^2 больше излучения, чем на нашей орбите? В 625 раз? Сколько там ватт на кв.метр спутники получают на орбите Земли?
Аноним 09/06/18 Суб 23:50:12  406399
>>406393
>Ты же понимаешь, что разница между орбитами
>>Януса и Эпиметея
>разница с пару сотен метров?
Серьёзно что ли?
Аноним 09/06/18 Суб 23:57:08  406402
>>406399
>Большая полуось 151 460 км
>Большая полуось 151 410 км
>Серьёзно что ли?
Нет, конечно
Аноним 09/06/18 Суб 23:58:38  406403
>>406399
50 км между орбитами, но по факту они не сближаются менее чем на 10000 км.
Аноним 10/06/18 Вск 00:04:19  406405
Как быстро и без смс вычислить угловой размер наблюдаемого объекта, если известно расстояние до него и размер?
Аноним 10/06/18 Вск 00:41:27  406413
>>406405
Арктангенс (размер/расстояние), но для малых углов можно просто размер/расстояние взять, разницы почти нет. Не забудь только из радиан в секунды дуги перевести, если надо.
Аноним 10/06/18 Вск 00:50:11  406416
>>406413
А если сильной точности и не требуется, то можно сразу в угловые секунды через формулу размер/расстояние*200000.
Аноним 10/06/18 Вск 01:29:20  406427
>>406416
А хорда, опирающаяся на угол меньший в 2 раза, тоже получается примерно меньше в 2 раза, правильно же? Опять же для малых углов, если длина хорды с википедии = 2 x R x sin(a).

Т.е. угловой размер (для малых углов) прямо пропорционален размеру объекта?

Если угловой размер Луны 32 минуты, то угловой размер булыгана с радиусом Луны/10 на таком же расстоянии будет в 10 раз меньше, около 3 минут?
Аноним 10/06/18 Вск 02:26:16  406433
>>406402
>>406403
> 50 км между орбитами, но по факту они не сближаются менее чем на 10000 км.
При том, что они малыши ~150 и ~100 км. А если землеподобные планеты? Думаю, так гравитационного влияния планет будет достаточно, чтобы при настижении более низокоорбитальной планетой более высокоорбитальной, наблюдаемый эффект произойдёт и с двумя планетами, заставляя их меняться местами. И, честно говоря, до сих пор не вижу обоснованных причин, почему не может быть системы, в которой планеты, настигая друг друга, меняются орбитами. Более того, чтобы смена орбита у планеты в такой системе проходила только с одной и той же планетой, а не ходила бы ближе-дальше.
Аноним 10/06/18 Вск 08:03:16  406436
>>406245
>пара светолет протяжённости галактик
...
Аноним 10/06/18 Вск 13:05:11  406453
>>404988 (OP)
Какой климат лучше: с низкими температурами или с высокими?

Вот у нас на Земле в умеренном климатическом поясе температура изменяется от ~-30 зимой до ~+30 летом при условии что комфортные температуры для человеков составляют от ~+10 до ~+30 не смотря на это с помощью технологических достижений мы более менее выдерживаем зимний период, а что если бы было наоборот? То есть зимой температура максимум падает до ~+10 а летом возрастает до ~+70 было бы человечеству легче переживать горячий вариант или наоборот сложнее? Наблюдали бы мы ситуацию вроде: Зимой всё цветёт, а летом всё вянет от жары и т.п.*
Аноним 10/06/18 Вск 13:12:07  406454
>>406453
Вопрос личных предпочтений
Аноним 10/06/18 Вск 13:40:48  406457
>>406453
Такс... ну тут у нас получилось бы следующее:
полярные территории стали бы умеренно холодными, их можно было бы заселять, на экваторе в этом случае была бы совсем йоба печка, скорее всего из за этого вменяемо даже не получилось бы плавать между двумя умеренными полюсами, так что путешествовали бы самолётами. На экваториальной суше тоже ловить не чего разве что солнечных батарей наставить, но вопрос кто в таких условиях их ставить будет.. При таких температурах океаны бы испарялись намного активнее, облачный покров на планете был бы в несколько раз плотнее, дожди были бы гораздо чаще и на большей территории, в целом атмосфера бы даже стала немного плотнее и насыщена влагой - это бы несколько сгладило климат между холодными частями и жаркими, но в целом было бы весьма душновато.. Для комфортной жизни пришлось бы поближе к полюсам засесть, в более жарких частях планеты в летнее время пришлось бы обмазываться радиаторами, охладителями и прочим в целом более сложным технологическим оборудованием нежели обогреватели. Основная жизнь кипела бы в ночное и утреннее\вечернее время, днём бы все старались свалить от дневного света куда подальше исключением разве что была бы облачная погода или если на территорию загонит холодные воздушные массы с полюсов. Касательно воздушных масс.. В настолько нагретой и чуть более плотной атмосфере йоба циклоны с антициклонами возникали бы на порядок чаще, всякие катаклизмы в лице ураганных ветров, смерчей и т.п. тоже возникали бы чаще и на большей территории, в общем было бы НЕПРИЯТНО . Как бы приличная часть людей совсем на полюса не перебрались, предпочитая контрить низкие температуры обогревателями зато при этом более спокойную часть атмосферы. Растительность была бы в большей степени адаптированная к высоким температурам, но в периоды лета вполне могла бы сбрасывать листву, разбрасывать семена до более холодных времён и т.п.

Кстати тут надо ещё смотря чем вообще вызваны высокие температуры, если в несколько раз более плотной чем земная атмосферой, то умеренные температуры могли бы установиться в целом повсюду с минимальными сезонными и суточными колебаниями, если речь о более горячей звезде, то тут может быть НЕПРИЯТНО больше не от температуры а от возросшей доли ультрафиолетового излучения в спектре звезды + более горячие звёзды заразы такие живут меньше. Если же речь о том, чтобы быть по ближе к звезде то тут аккуратно надо, а то и тидально локнуть может, ну или как минимум сильно замедлить вращение планеты вызвав более острые суточные колебания. Если же нагрев вызван воздействием приливных сил от газового гиганта\парной планеты\спутника на умеренно низкой орбите, то тут вообще могут появиться регулярные йоба планетотрясения что испортят жизнь по больше чем радиация и температура, в случае газового гиганта же можно и радиации от него дофига нахватать. Ну примерно так.
Аноним 10/06/18 Вск 14:24:41  406463
>>406395
>>406427

бамп
Аноним 10/06/18 Вск 15:07:29  406464
Во-первых, Солнце - это планета! Во-вторых, откройте энциклопедию по физике, найдите "Солнце". Обратите внимание на ускорение свободного падения на поверхности Солнца =257 м/с^2 То есть Солнце, согласно научным данным - чёрная дыра. Тогда как происходит выброс плазмы при таком ускорении? На каком расстоянии от поверхности Солнца надо разместить теннисный мяч, чтобы Солнце по теории Эйнштейна исчезло? Температура на Солнце - это фазовое состояние атомной структуры. Есть фотографии НЛО (или материального тела), который спокойно летает вблизи поверхности Солнца.
Аноним 10/06/18 Вск 15:22:08  406466
>>406464
Ну пиздец это не то что не космоопера и даже не космофентези это уже воистину мир бреда и абсурда, 10 из 10 \b\++...
Аноним 10/06/18 Вск 20:40:13  406518
Господа, у меня параноя или последние скажем так 10 лет в атмосферу планеты слишком часто начали падать астероиды?
Возможно это связано с распространением камер, просто стали чаще на записи попадать регулярные для планеты события
Аноним 10/06/18 Вск 21:19:05  406524
>>406518
>у меня параноя
this. Астероиды уже 65 миллионов лет не падали
Аноним 10/06/18 Вск 21:28:19  406526
>>406524
Абсолютно точный и абсолютно бесполезный ответ.
Все время забываю что надо искать ответы самому а не спрашивать анонимных уёбков в сети.
Спасибо что напомнили, добра вам )
Аноним 10/06/18 Вск 21:31:44  406527
>>406526
Иди с миром и не трать наше время больше. И таблетки прими.
Аноним 10/06/18 Вск 22:02:25  406535
>>406524
Уебок, съеби с этой доски
Аноним 10/06/18 Вск 22:04:09  406537
>>406524
Але, маня, астероиды падают каждый год, или у тебя есть какое-то свое собственное определение?
Аноним 10/06/18 Вск 22:58:57  406544
>>406524
>>406526
>>406535
>>406537
Девчата, не ссорьтесь, только тред засрали почём зря. Астероид это шелупонь от 1 метра и до здоровенной хуйни под 1000 километров в диаметре.
Аноним 10/06/18 Вск 23:02:43  406545
Хочу зайти в ракетное моделирование с перспективой поднять пепелац до 100км+ Тех опыта за плечами нет, в каком порядке курить литературу?

Есть место под полигон, без людей, там даже и ПВО то нет, даже никто и не узнает.
Аноним 10/06/18 Вск 23:08:13  406547
>>406545
Начни с административного кодекса (если ты из этой страны). Пиротехнические составы в военном деле, пороходелие, теория летательного аппарата. Кури форумы карамельщиков - полезно. Чуть пониже есть /spc prgrm тред, там немного литературы вбрасывали
Аноним 10/06/18 Вск 23:10:02  406548
>>406547


А если у меня есть знакомые мужики которые могут подогнать цистерну жидкого кислорода?
Аноним 10/06/18 Вск 23:14:04  406551
>>406548
Жидкостный двигатель очень сложно сделать в гараже, только петарды. А им кислород не очень нужен.
Аноним 10/06/18 Вск 23:14:45  406552
>>406548
Ты всё равно не сможешь его хранить. И учти, что ЖРД на порядок(ки) сложнее твердотопливного. (Хотя в вышеозначенном треде говорили, что АК не ограничивает запуски ЖРД)
Аноним 10/06/18 Вск 23:16:11  406554
>>406551
Ну тащемта можно запилить гибридный двигатель а-ля Рутан, но это будет всё равно сложнее, чем чисто твердотопливный
Аноним 10/06/18 Вск 23:17:20  406555
>>406551


>Жидкостный двигатель очень сложно сделать в гараже

Че там сложного если не мудрить с нагнетателем и нагнетать батареечкой как одни умные люди? 150 атмосфер и не надо, 50 нагнать - уже неплохо а болванку камеры и сопла васьки-токарщики сделают.
Аноним 10/06/18 Вск 23:18:04  406556
>>406555
>нагнетать батареечкой
Щито?
Аноним 10/06/18 Вск 23:18:52  406557
>>406556


Ракета электрон.
Аноним 10/06/18 Вск 23:24:04  406560
>>406556
Для подачи в камеру сгорания топлива и окислителя используется два электронасосных агрегата (ЭНА) мощностью 37 кВт[2][4][5]. Поскольку каждый компонент подаётся собственным насосом, расход и соотношение компонентов регулируются изменением расхода насосов, а не с помощью дросселя и регулятора как в традиционном ЖРД с использованием турбонасосного агрегата[1]. Такая схема управления двигателем впервые использована на действующей ракете-носителе[6].

Каждый ЭНА приводится в действие бесщёточным электродвигателем переменного тока, питающимся от установленных на ступени литий-полимерных аккумуляторных батарей через инвертор. Максимальная частота вращения центробежного насоса составляет 40 000 об/мин, что позволяет изменять давление в топливной магистрали от 0,2 до 20 МПа (от 2 до 200 кгс/см²
Аноним 10/06/18 Вск 23:24:25  406561
>>406557
Мог бы и сам рассказать. Вообще, аккумуляторы для обслуживания нужд ЖРД - зашквар. Может, топливные элементы вышли бы компактнее, но всё равно многократно тяжелее турбонасоса
Аноним 10/06/18 Вск 23:26:06  406564
>>406561

>Вообще, аккумуляторы для обслуживания нужд ЖРД - зашквар.

https://www.youtube.com/watch?v=eg5234BOED8

Летает вроде.
Аноним 10/06/18 Вск 23:28:03  406565
>>406564
С лёгким ТНА летало бы быстрее, выше, сильнее. Хотя если делать дёшево и сердито - сойдёт. Интересно, где они взяли дмгатели на 40000 оборотов
Аноним 10/06/18 Вск 23:44:56  406573
>>406565

На али купил, хуле ты.
https://www.alibaba.com/product-detail/16mm-12v-electrical-brushless-DC-motor_60613878945.html?spm=a2700.7724857.main07.30.1180aaf5iZv1gi
Аноним 11/06/18 Пнд 00:33:30  406600
videoplayback.webm (21294Кб, 640x360, 00:04:58)
>>406555
Да вообще без проблем, хули там собирать
Аноним 11/06/18 Пнд 10:27:34  406627
>>406600


попилы-откаты
Аноним 11/06/18 Пнд 11:32:57  406629
>>406561
Тем не менее рокетлеб пока единственные, кто получил у керосинового двигателя УИ на уровне моря за 300 не закрытым циклом
Аноним 11/06/18 Пнд 11:36:49  406630
>>406600
Да, меня это очень забавляет. Казалось бы, первые российские частники, пилящие ракеты. А у них же на сайте в новостях
> Несмотря на финансовые и административные сложности мы уверенно продвигаемся вперед к светлому космическому будущему для всего человечества!
Аноним 11/06/18 Пнд 11:40:58  406631
Да, кстати, насчёт ЖРД. Я тут посмотрел, у азота температура кипения -195°C, у ацетилена -83°C. Получается, что с помощью жидкого азота вполне можно получить жидкий ацетилен. Какие тут подводные?
Аноним 11/06/18 Пнд 11:57:00  406632
>>406631
Ты хоть представляешь себе, какие меры приходится принимать для того, чтобы даже газообразный ацетилен не бомбанул? Безо всяких причин, просто потому ч тоему так хочется. А уж о том, чтобы сжимать его и речи не идёт
Аноним 11/06/18 Пнд 12:01:42  406633
>>406632
Зачем сжимать? 25 атмосфер в стандартном сварочном баллоне должно хватить более чем. Если газообразный ацетилен пропускать через тонкую трубку, охлаждаемую ниже температуры кипения, то это должно сработать. Утечки это другой вопрос
Аноним 11/06/18 Пнд 12:03:46  406634
>>406633
Я накачу за тебя, когда ты взлетишь на воздух
Аноним 11/06/18 Пнд 12:06:36  406635
>>406634
Пусть это теоретически будет в атмосфере почти без кислорода, азотной допустим. Сам принцип то работать будет?
Аноним 11/06/18 Пнд 12:42:17  406638
Можно ли приспособить верхнюю ступень протона под ещё более опасное топливо?
Аноним 11/06/18 Пнд 13:43:18  406646
>>406638
новая топливная смесь = новый двигатель = новая ступень
Аноним 11/06/18 Пнд 14:00:54  406649
>>406646
Не всегда же.
Аноним 11/06/18 Пнд 15:29:28  406653
>>406630
А что ты хочешь, если можно набутылиться за сахар?
Аноним 11/06/18 Пнд 15:31:25  406654
>>406633
Нюфаня, ацетилен в сварочном баллоне находится не в сжатом, а в растворённом виде. Попробуй как-нибудь приподнять "пустой" ацетиленовый баллон, лолкек.
Аноним 11/06/18 Пнд 16:10:46  406658
>>406654
В сжатом, но с абсорбером.
Аноним 11/06/18 Пнд 16:17:00  406660
>>406561
Но ведь аккумуляторы это разновидность топливных элементов.
11/06/18 Пнд 16:38:55  406664
>>406629
Мечтай.
>>406649
А как иначе-то.
Аноним 11/06/18 Пнд 16:46:21  406665
>>406664
https://ru.wikipedia.org/wiki/%D0%A0%D0%B5%D0%B7%D0%B5%D1%80%D1%84%D0%BE%D1%80%D0%B4_%28%D1%80%D0%B0%D0%BA%D0%B5%D1%82%D0%BD%D1%8B%D0%B9_%D0%B4%D0%B2%D0%B8%D0%B3%D0%B0%D1%82%D0%B5%D0%BB%D1%8C%29?wprov=sfla1
> Удельный импульс Вакуум: 333 c
> Ур. моря: 303 c
Аноним 11/06/18 Пнд 16:48:52  406666
>>406664
Ну навскидку могу привести в пример двигатель агены.
11/06/18 Пнд 17:04:04  406668
>>406665
>ru.wikipedia.org
мда
>>406666
Поменяли смесь азотной кислоты и тетраоксида азота на смесь азотной кислоты и тетраоксида азота, ну.
Аноним 11/06/18 Пнд 17:34:22  406672
>>406666
Титан еще. Перевели с керосинки на НДМГ.
Аноним 11/06/18 Пнд 18:51:07  406687
Какой предел размеров и мощности для петард?
Аноним 11/06/18 Пнд 19:40:19  406692
>>406687
Самую большую вроде Aerojet делали, диаметром 260 дюймов (6,6 метров), и она даже кое-как работала на наземных тестах, но до полетов не дошло. Может, и больше можно, хуй знает.
Аноним 12/06/18 Втр 00:35:24  406706
>>406660
Не совсем. Скорее сходство имеется с неперезаряжаемыми гальваническими элементами, но ТЭ имеют внешний источник реактивов а не ограничены запиханным в них при производстве количеством. Да и реактивы там совсем другие
Аноним 12/06/18 Втр 02:08:19  406713
>>406254
Джуно, тащемто, тоже солнечные панели имеет, а он вокруг Юпитера летает, лел.
Аноним 12/06/18 Втр 09:42:54  406726
>>406713
Хуевое сравнение, даун летал на ионниках с запиткой от панелей, а у джуно гептилка.
Аноним 12/06/18 Втр 16:19:30  406760
Чем закончилась история Звезды Табби?
достроили они там свою теплицу?
Аноним 12/06/18 Втр 17:20:37  406772
>>406760
Естественное происхождение наиболее вероятно.
Аноним 12/06/18 Втр 18:10:27  406790
Насколько сложно организовать курилку на МКС?
Аноним 12/06/18 Втр 19:08:20  406800
>>406790
Да в принципе несложно по сравнению c тем, что там уже есть. Но зачем?
Аноним 12/06/18 Втр 19:26:33  406803
357439677.jpg (19Кб, 268x265)
>>406800
Чтобы курить.
Аноним 12/06/18 Втр 19:27:37  406804
А нельзя в прогрессе курить, например?
Аноним 12/06/18 Втр 20:19:36  406813
>>406804
Он же махонький, вангую что там от собственного дыма через минуту прихуеешь.
Аноним 12/06/18 Втр 21:00:27  406818
>>406813
>>406804
>>406790
В форточку кури
Аноним 12/06/18 Втр 21:21:47  406822
>>406818
Артемьев жалуется, что у него от частой разгерметизации уши болят.
Аноним 12/06/18 Втр 21:47:03  406825
>>406822
Ну так пускай ватку проспиртованную засунет.
Аноним 12/06/18 Втр 21:54:52  406826
>>406825
Я когда ему твой совет передал, он эту ватку мне пообещал засунуть. Куда не сказал, но я в принципе догадываюсь.
Аноним 12/06/18 Втр 22:18:57  406828
>>406822
В американский модуль иди
Аноним 12/06/18 Втр 22:46:24  406831
>>406790

Ну дым понятно. Поставить вытяжку и пропускать через неё. А вот что с пеплом делать это сложнее вопрос.
Аноним 12/06/18 Втр 22:57:16  406833
>>406831
Ну, если на МКС срут в пылесос, то и пепел в пылесос стряхивать решение очевидное.
Аноним 12/06/18 Втр 23:38:06  406839
>>406726
Об ионниках вообще разговора не было.
Аноним 13/06/18 Срд 09:34:20  406877
>>406772
Давай подробности
Аноним 13/06/18 Срд 12:22:32  406885
>>406658
Ладно.
>...Допускать в ацетиленовых баллонах давление, значительно превышающее 25 кг/см2, нельзя по условиям безопасности. Вследствие этого пористую массу пропитывают ацетоном, который существенно повышает газовбираемость, так как является хорошим растворителем для ацетилена.  
Аноним 13/06/18 Срд 13:07:48  406889
В чем разница между гидростатическим равновесием и просто круглостью?
Аноним 13/06/18 Срд 13:19:21  406890
>>406889

Если планета из однородной жидкости вращается, то она уже не будет шариком.
Аноним 13/06/18 Срд 17:16:55  406918
Какие были планы на Энергию-Буран, Энергию-2 и Вулкан?
Аноним 13/06/18 Срд 17:58:36  406920
>>406918
Размещение оружия в космосе.
13/06/18 Срд 21:03:08  406934
>>406918
В сущности-то не было.
Аноним 13/06/18 Срд 21:10:05  406937
Если титану сделать содержание кислорода как на земле, там всё ебанет?
Аноним 13/06/18 Срд 23:18:56  406959
>>406918
Смотрите, пиндосы челнок делают! Мы тоже хотим!
Аноним 14/06/18 Чтв 15:56:36  407038
>>406937
Нет, кислород сконденсируется и не сможет вступить в реакции при такой температуре и давлении.
Аноним 14/06/18 Чтв 20:36:05  407066
Вилка.jpg (82Кб, 1024x768)
Тут недавно проскакивала новость, что Оппортьюнити перевели в ждущий режим, чтобы не тратил энергию, из-за пылевой бури, которая засрала ему солнечные панели. А после бури как он их чистить-то будет?
Аноним 14/06/18 Чтв 20:42:26  407067
>>407066
Ну вообще буря закрыла все небо и там темнее чем ночью, засерание вторично. В прошлый раз пыль сама сдулась, например.
Аноним 14/06/18 Чтв 21:40:15  407069
Кстати почему ему не приделали дворники или вентилятор?
Аноним 14/06/18 Чтв 22:01:50  407072
>>407069
У него миссия блять на 90 дней была рассчитана, кто ж мог подумать что он там 14 лет будет бури превозмогать.
Аноним 14/06/18 Чтв 22:20:22  407073
interstellar-ca[...].jpg (63Кб, 1024x578)
А какова хуя черные дыры вращаются?

На их горизонте время для внешнего наблюдателя уже бесконечно замедлено, почему тогда жидовские ученые говорят, что какая-то хуйня внутри вращается на бешеных оборотах?
Аноним 14/06/18 Чтв 22:42:41  407077
>>407073
Извинись за жидовских ученых сначала, а потом получишь ответ.
Аноним 14/06/18 Чтв 22:55:00  407079
>>407073
>внешнего наблюдателя
А причем тут внешний наблюдатель-то вообще? Дыра вращается вне зависимости от его наличия или присутствия. Или "если я не вижу луну, значит ее нет"?

Дыра вращается потому что момент импульса сохраняется. Если она, например, получилась путем коллапса массивной звезды, то для начала представь, что звезда вращалась. Не очень быстро. Потом после коллапса часть звезды стала ЧД. И эта ее часть продолжила вращаться. А чо б ей не продолжить.
только выиграли даже ускорилась.

Как фигуристка ускоряет вращение, когда руки к сиськам поджимает.
Аноним 14/06/18 Чтв 23:14:18  407083
>>407079
Тогда почему нам есть какое-то дело до этого? Мы же внешний по отношению к черной дыре наблюдатель, и никакой эргосферы быть не должно.
Аноним 14/06/18 Чтв 23:21:57  407085
>>407083
Ну черная дыра не вечный объект же, ну, они испаряются когда-то. Когда-то какие-то эффекты из эргосферы могут возыметь наблюдаемый эффект, при каких-то условиях.
Аноним 14/06/18 Чтв 23:26:53  407086
>>407083
>Мы же внешний по отношению к черной дыре наблюдатель, и никакой эргосферы быть не должно
Блеать, я не понимаю как в твоей логике связаны наличие или отсутствие внешнего наблюдателя и наличие или отсутствие эргосферы.

Кто тебе сказал, что наблюдатель на что-то влияет? Эргосфера у вращающейся ЧД есть не потому за ней кто-то наблюдает или не наблюдает.
Аноним 14/06/18 Чтв 23:31:23  407088
Если вращение дыры неважно для окружающего мира, от чего зависит направления вращения аккреционного диска?
Аноним 14/06/18 Чтв 23:33:28  407089
Какая скорость объектов на границе наблюдаемой вселенной относительно нас?
Аноним 14/06/18 Чтв 23:37:41  407090
>>407088
Зависит от природы аккреционного диска. Если это остатки говна, которое вращалось от звезды, которая когда-то стала черной дырой, то это вращение будет преобладать. Но черные дыры ведут активную сексуальную жизнь в звездных парах, так что там может появиться такой партнер, который по ретроградке будет двигаться к анусу черной дырочки и потом с него одежду также будут по ретроградке снимать...
Аноним 14/06/18 Чтв 23:46:51  407091
>>407088
>вращение дыры неважно
Говну всегда что-то "неважно". g:"метрика Керра"
Аноним 14/06/18 Чтв 23:50:31  407092
У нас есть человек в скафандре с рюкзаком. В рюкзаке находится некий массивный объект, который человек запускает в строго одном направлении придавая себе ускорение. Запускает он его среднестатистическим броском рукой, с поправкой на то что он очень точен, и кидает предметы в строго одном направлении, со строго одинаковой силой.
Как лучше всего придавать себе импульс если кидать по 1 грамными шариками или сразу оттолкнутся от 1 тонной конструкции?
Аноним 15/06/18 Птн 00:37:17  407093
>>407092
Ну, чем выше скорость рабочего тела, тем эффективнее движение.
Аноним 15/06/18 Птн 03:15:46  407098
canal.jpg (108Кб, 1600x908)
26568851319.jpg (211Кб, 1920x1080)
196701888.jpg (4779Кб, 2880x3600)
>>406692
Ебать громадина!

https://youtu.be/rmtzFNy1t3U
Аноним 15/06/18 Птн 04:12:14  407101
>>407093
Ну и где будет выше скорость? Если оттолкнутся от 1000 кг тела или от 1 кг тела тысячу раз?
Аноним 15/06/18 Птн 09:41:32  407123
4514688735c0635[...].jpg (1809Кб, 3036x2272)
Aerojet260sl2.jpg (81Кб, 443x667)
>>407098
Если кому интересно, он после испытаний с видеорилейтед до сих пор в этой шахте лежит.
Аноним 15/06/18 Птн 11:31:35  407149
>>406877
Там карочи плонита, оче большая плонита, прямо писец короче, и вокруг этой планеты оче очебельные йоба кольца, прямо огромнейшие до невозможности и они короче и закрывают свет, а ещё там оче толстый и богатый на астероиды пояс астероидов это ещё дополнительно форсит мемес отсутствие фотонов.
Аноним 15/06/18 Птн 12:53:55  407159
>>407101
1000 от 1 кг, ты чего? Очевидно, что бросить 1 кг шарик получится с большей скоростью, импульс - масса х скорость. Анон выше же написал.
Аноним 15/06/18 Птн 12:56:40  407161
>>407123
По этим фото не скажешь, что 4 человека должны друг другу на плечи встать чтобы сравнять размеры.
Аноним 15/06/18 Птн 13:30:54  407166
изображение.png (2841Кб, 1920x1080)
Ой хватит мне сказки за вечность заяснять, я думаю ты жестянка даже число Грэма лет не проживёшь.
Аноним 15/06/18 Птн 13:42:47  407167
>>407166
>Скакал углеродоущерб неосиливающий сохранить сложную структуру даже две сотни лет.

Как тебе постоянные процессы отмирания тканей и попыток пожрать тебя изнутри "симбиотической" микрофлоры?
Аноним 15/06/18 Птн 13:47:51  407169
>>407167
Ай, это же не тот тред, глубочайше извиняюсь за каку.
Аноним 15/06/18 Птн 14:15:53  407174
>>407073
Потому что момент импулса.
Аноним 15/06/18 Птн 17:21:44  407195
Как работает магнитное поле? Появляется разница в заряде между двумя объектами, отрицательно заряженные частицы начинают перескакивать от "минуса" к "плюсу", правильно же? Если поставить стенку - частицы не перескакивают, всё, пизда полю.
Свет, радиоволны - поток определенных частиц. Поставил стенку - ничего не проходит.
Радиоактивное излучение - поставил стенку, ничего не проходит.

Что является "переносчиком" гравитационного взаимодействия? Школьная физика и википедия говорит, что сила гравитации подчиняется закону обратных квадратов, что говорит о том, что переносчиками гравитации являются какие-то частицы. Откуда берется энергия и такое количество "переносчиков" на это постоянное и бесконечное взаимодействие между всеми объектами и даже элементарными частицами? Почему эти частицы не взаимодействуют друг с другом, как такое вообще может быть? Почему вдруг от гравитации нельзя спрятаться, почему это вдруг сила гравитации не уменьшается при воздействии через другие объекты, такие как миллион тонн породы Земли между бухим в салат аноном в пятницу и ядром планеты?

Не надо кидать сюда ссылки на научные работы в 300 томов, если анон не может объяснить простым и доступным читателю языком - значит он сам нихуя не знает. Может мы все живем в Матрице, а?
Аноним 15/06/18 Птн 17:47:11  407196
>>407195
Ну это бля гравитационные волны короче
Аноним 15/06/18 Птн 17:49:45  407197
>>407159
Ну и что будет иметь большую скорость?
Запуск тысячи килограмовых шаров или запуск одного 1 тонного шара?
Аноним 15/06/18 Птн 17:53:51  407198
>>407197
Какую скорость ты можешь придать шару в 1 грамм и в 1 тонну?
Аноним 15/06/18 Птн 19:16:41  407204
>>407092
Тут хак в том, что ты будешь 10^6 раз бросать шарики в 1 грамм с ускорением в 30 м/с^2 и той же силой, с которой всего 1 раз оттолкнешь от себя 1 тонный шар с 0.1 м/с^2.

Если, например, заменить твою руку на манупилятор, причем сила затраченная на бросок будет строго пропорциональна разряду батареи и стоит задача бросить все шарики, то тогда сила приложенная к шарику в 1 грамм будет в 10^6 раз слабее и разницы никакой не будет.
Аноним 15/06/18 Птн 19:45:38  407206
А вот стандартная пара кислород + керосин.

насколько падает КПД если в камере сгорания не 150 атм а 50? или вообще фокуса не происходит? или нужно менять пропорцию?
Аноним 15/06/18 Птн 21:34:03  407208
У какой ракеты самая хуевая аэродинамика?
Аноним 15/06/18 Птн 22:05:50  407209
Ricerocket6.jpg (31Кб, 450x321)
>>407208
У ракеты с квадратным обтекателем, я полагаю.
Аноним 15/06/18 Птн 22:44:05  407213
>>405092

Они скорее всего и есть ошибочные, так же как представления средневековых врачей или алхимиков.
Аноним 15/06/18 Птн 23:28:02  407224
>>405141

От перегрузок на старте у них губы лопаются. антиквартная шутка из 80-х
Аноним 15/06/18 Птн 23:57:36  407230
>>407196
Свет - это тоже волна, но свет не умеет проходить метр бетона. Почему гравитация может?
Аноним 16/06/18 Суб 00:27:17  407236
>>407230
Ну нейтрино через что угодно проходят, например.
Аноним 16/06/18 Суб 03:13:26  407266
>>407195
>Что является "переносчиком" гравитационного взаимодействия?
Ты это на борде спрашиваешь? Лучшие умы человечества, бьются над разгадкой уже столетие, коллайдер построили и ещё толком не поняли как и почему. Если узнаешь, что является переносчиком, то нобелевка, да какая нахуй нобелевка, вообще всё что угодно тебе будет обеспечено.
Аноним 16/06/18 Суб 03:28:04  407271
>>407236
>>407266
Жидовская наука какая-то.

А как нейтрино проходит? Как оно?он? не сталкивается с хуллиардом других частиц по пути? И почему тот же фотон не может так же?

Почему они не теряют скорость во время огибания препятствий?
Аноним 16/06/18 Суб 08:21:32  407281
>>407271
У тебя есть шанс начинать узнавать это с сентября, слушая учителя, а не пинеая хуи на двачах
Аноним 16/06/18 Суб 11:20:08  407286
Анэны, есть ли фотографии марсоходов сделанные другими марсоходами?
Аноним 16/06/18 Суб 11:24:47  407287
image.jpeg (519Кб, 1039x840)
>>407286
Аноним 16/06/18 Суб 11:27:15  407288
>>407286
Нет, какой смысл сажать их в одинаковое место?
Аноним 16/06/18 Суб 11:28:18  407289
>>407287
Судя по гуглу, это селфи какого-то марсохода после какого-то бурения, но не фото марсохода сделанное другим марсоходом.
Есть трустори фото?
Аноним 16/06/18 Суб 11:29:43  407290
>>407288
Шоб сделать красивое фото, очевидно же
Аноним 16/06/18 Суб 11:36:25  407292
>>407287
Я чет не пойму где палка за которую держится развёрнутая на марсоход камера? Или уже технологии антигравитации\тонкого контроля магнитных полей освоены?
Аноним 16/06/18 Суб 11:38:03  407293
>>407067
>В прошлый раз пыль сама сдулась, например.
И после этого атеистобляди смеют утверждать что Бога нет, ясно.
Аноним 16/06/18 Суб 11:39:09  407294
>>407293
Его и нет тут у нас ТВЁРДАЯ реальность, так что только загоны физики в данном случае в лице ветра.
Аноним 16/06/18 Суб 11:43:11  407295
>>407294
Ты действительно такой тупой и не понимаешь, что ветер в разреженной атмосфере Марса не мог случайно появиться в определенный момент?
Мда, ну ты и скептик.
Аноним 16/06/18 Суб 11:45:05  407296
Чому чёрные дыры не могут увидеть, если вокруг них должны быть ебические аккреционные диски?
Аноним 16/06/18 Суб 11:48:52  407297
latest[1] (275Кб, 483x620)
>>407295
Будь реальность не твёрдой мы бы уже псионическую теорию изучали и в Покров лезли. Но вместо этого только на ракеты водой брызгаем.
Аноним 16/06/18 Суб 11:51:14  407298
>>407297
Зачем споришь с религиозным дураком?
Аноним 16/06/18 Суб 11:52:33  407299
>>407298
Да так, "просветить хотел".
Аноним 16/06/18 Суб 11:56:21  407300
>>407296

Ну наблюдений рентгеновского излучения оттуда достаточно много.
Аноним 16/06/18 Суб 11:58:25  407301
>>407290
Только аполлоны вроде садились рядом с сурвейором и делали фоточки, но это потому что сурвейоры должны были искать место посадки.
>>407292
Фото из многих кусков потому что.
Аноним 16/06/18 Суб 12:00:46  407303
1529139622376-6[...].jpg (3212Кб, 2048x1793)
>>407301
Аноним 16/06/18 Суб 12:03:48  407304
>>407301
> Только аполлоны вроде садились рядом с сурвейором и делали фоточки, но это потому что сурвейоры должны были искать место посадки.
А фото нет в открытом доступе?
Гуглю на ангельском, ничего не могу найти.
Аноним 16/06/18 Суб 12:08:50  407306
>>407304
Бля, я тупой.
Я думал речь идет о Марсе, но на самом деле о Луне >>407303
Аноним 16/06/18 Суб 12:47:03  407312
>>407296
Аккреционные диски тебе ничего не должны
Аноним 16/06/18 Суб 12:51:56  407315
Че там кстати с фоточками чёрной дыры в центре? Я уже начинаю подозревать что это мошенничество.
Аноним 16/06/18 Суб 12:52:54  407316
Что будет если завести на марс растения и микробы, толерантных к тамошней среде?
Аноним 16/06/18 Суб 13:39:30  407324
>>407316
На марсе будут растения и бактерии, толерантные к его среде.
Аноним 16/06/18 Суб 14:03:09  407326
39.jpg (32Кб, 620x370)
>>407324
Не может быть
Аноним 16/06/18 Суб 15:09:38  407344
>>407281
> есть шанс
Поздно уже, нету. Последняя надежда на двачи
Аноним 16/06/18 Суб 19:01:24  407382
>>407344
Есть четыре фундаментальных взаимодействия — сильное, слабое, электромагнитное и гравитационное, причем каждое последующее слабее предыдущего на несколько порядков. Сильное и слабое еще и относятся к короткодействующим, т.е. убывают с расстоянием сильно быстрее, чем по закону обратных квадратов.

Чем в меньшем количестве взаимодействий участвует частица, и чем более слабы эти взаимодействия, в которых она участвует, тем более ей похуй на материю и тем свободнее она проходит через что угодно.

Нейтрино, например, участвует только в слабом и гравитационном взаимодействии, да еще и имеет ничтожно малую массу, поэтому ему практически полностью поебать на любое вещество и оно легко пролезет без всякого мыла прямо через атомное ядро. Слабое взаимодействие, как и следует из названия, не шибко сильное, и действует на очень малых расстояниях, а гравитация начинает заметно влиять на траекторию нейтрино только начиная с поистине астрономических масс.

Гипотетический переносчик гравитации, если и существует, то скорее всего не участвует вообще ни в каком взаимодействии, помимо гравитационного, проходит насквозь вообще через все, и задетектить его вообще хуй знает как.
Аноним 16/06/18 Суб 20:17:05  407395
>>407382
А какое отношение к этому имеют гравитационные волны и ЛИГО?
Аноним 16/06/18 Суб 20:47:13  407396
>>407395
Никакого, это не частицы и не переносчики гравитации.
Аноним 16/06/18 Суб 21:06:23  407397
Не знаю тупой вопрос или нет, в общем как попасть на работу в обсерваторию?
Где лучше учиться, на какой специальности. Можно ли попасть в обсерваторию без соответсвующего образования? Какие там нужны специалисты, непосредственно астрономы, электронщики, радиофизики и т. д. Если у меня уже есть высшее образование не по профилю (в данном случае инженер ПГС), можно ли в таком случае туда попасть? Сложно ли это сделать?
Аноним 16/06/18 Суб 21:43:30  407400
>>407382
Спасибо
Аноним 17/06/18 Вск 04:00:56  407434
>>407312
Они мне излучение должны.
Аноним 17/06/18 Вск 10:37:36  407470
>>407434
Чё, и долговая расписка есть?
Аноним 17/06/18 Вск 11:27:51  407475
>>407470
Ага, зверенная Максвеллом и Эйнштейном.
Аноним 17/06/18 Вск 12:20:32  407489
>>407286
Если бы такое фото было, то это бы значило то, что ученые отправили два марсохода, стоимостью пару лярдов долларов, исследовать одно и то же место, что в принципе является глупостью.
Аноним 17/06/18 Вск 13:02:41  407496
>>407382
Слабое и электромагнитное - одно и то же электрослабое взаимодействие, т е не четыре, а три
Аноним 17/06/18 Вск 13:15:16  407497
>>407496
Так можно и до теории всего и единого взаимодействия допиздеться, обычно рассматривают все же четыре.
Аноним 17/06/18 Вск 13:15:47  407498
>>407496
Олдфаг в треде, все в сингулярность!
Аноним 17/06/18 Вск 15:38:58  407505
Помогите опознать блок маневровых двигателей плиз
https://twitter.com/runnymonkey/status/1008323221874233344
Аноним 17/06/18 Вск 15:51:47  407506
blocke.png.jpg (49Кб, 490x388)
85671-i093.png (40Кб, 404x450)
>>407505
Больше всего похоже на двигатели Блока Е, а не Г, пикрилейтед.
http://www.yuzhnoye.com/company/history/block_e.html
17/06/18 Вск 16:35:15  407507
SNC16397.JPG (238Кб, 1030x773)
SNC16398.JPG (237Кб, 1030x773)
>>407505
На нём же ж написано - 11Д71 - блок ДУ СОЗ блока Д.
Аноним 17/06/18 Вск 17:13:50  407512
>>407507
>>407506
Спасибо большое.
Аноним 17/06/18 Вск 17:17:52  407513
Что произойдет с планетой, если ее масса (не объем) изменится хотя бы на 1%?
Аноним 17/06/18 Вск 17:19:55  407514
>>407507
Там этому мужику уже кто-то в твитторе твой ответ передал если что.
Аноним 17/06/18 Вск 18:32:38  407517
>>407513

Особо ничего. А как ты себе это представляешь?
Аноним 17/06/18 Вск 19:17:04  407523
>>407517
Ну, сместится ли она с орбиты, изменится ли ее скорость вращения вокруг своей оси (или вокруг солнца)?
Аноним 17/06/18 Вск 19:28:34  407526
>>407523

Это зависит от изменения импульса и момента импульса. Если импульс останется прежним, то станет чуть ближе летать к светилу и чуть медленнее вращаться.
Аноним 17/06/18 Вск 19:48:15  407529
>>404988 (OP)
Есть ли в космосе ещё какие йобы, типа черной дыры? Чтоб необычно так было и захватывающе.
Аноним 17/06/18 Вск 19:56:51  407530
237190611f92a82[...].jpg (25Кб, 320x240)
>>407526
Спасибо, анон
Аноним 17/06/18 Вск 19:59:50  407532
>>407529
Кефир
Аноним 17/06/18 Вск 20:49:29  407539
>>407529

Тёмная материя, тёмная энергия, квазары, скопления чёрных дыр, гамма-всплески, двойные звёзды. Войды.
Аноним 17/06/18 Вск 20:56:12  407540
>>407539
>Войды
Можно подробнее.
Аноним 17/06/18 Вск 20:59:28  407542
>>407540
Огромные пространства где ниче нет.
Аноним 17/06/18 Вск 20:59:44  407543
>>407540
Очень дохуя пустоты. На пикиведии же:
https://ru.wikipedia.org/wiki/Войд
Аноним 17/06/18 Вск 21:45:39  407546
Зачем у посадочных аппаратов с Венеры были тормозные парашюты?
Аноним 17/06/18 Вск 22:08:22  407549
>>407546
Чтобы подольше пробыть в верхних слоях и насобирать инфы о том как там оно, насколько я помню.
Аноним 18/06/18 Пнд 22:02:59  407647
igry-novosti-20[...].jpg (25Кб, 644x362)
>>407540
Ставь купол на как можно большее количество вражин, прожимай мом и манту и еби в ротешник всех кого можешь
Аноним 19/06/18 Втр 06:25:20  407684
а большой взрыв на самом деле не взрыв потому, что не детонировал?
Аноним 19/06/18 Втр 08:06:53  407685
>>407684
Это был Большой Пук.
Аноним 19/06/18 Втр 13:55:47  407712
>>407684
Не всегда взрыв сопровождается детонацией
Аноним 19/06/18 Втр 15:07:35  407716
15292551059320.jpg (127Кб, 658x960)
Кто сказал, что раньше Земля была меньше? За счёт чего же она тогда сейчас выпучилась?
Если так, то как же тогда гравитация работала?
Аноним 19/06/18 Втр 15:16:50  407718
Каким образом Арес 5 должен был тащить 180 тонн?
СЛС по факту мощнее Ареса, более эффективные маршевые двигатели и ускорители, но в максимальной конфигурации обещают 130-150 тонн
Аноним 19/06/18 Втр 15:21:48  407719
>>407718
RS-68 раза в полтора мощнее RS-25, на верхней ступени J-2 в десять раз мощнее одного RL-10.
Аноним 19/06/18 Втр 15:24:49  407720
>>407719
Но у rs25 больше УИ.
А какой импульс у rl 10? Я не нашел
Аноним 19/06/18 Втр 15:27:21  407722
>>407720
Компенсируется большей массой топлива. У RL-10 импульс 465, лучший среди химических, против 440 вроде у J-2X.
Аноним 19/06/18 Втр 15:43:16  407727
>>407716
Кто тебе сказал что Земля была меньше?
Аноним 19/06/18 Втр 15:48:32  407728
>>407727
Научпопные фильмы про Пангею
Аноним 19/06/18 Втр 16:15:02  407730
>>407722
>лучший среди химических
Ты забыл про КВД-1
Аноним 19/06/18 Втр 16:15:25  407731
>>407716
Да так же она работала, в пределах долей процентов, большее влияние Луна оказывала, так как ближе была.
Аноним 19/06/18 Втр 16:22:17  407732
>>407730
Нет ты.
Импульс 462 против 465, про применение и говорить нечего.
Аноним 19/06/18 Втр 16:24:20  407734
>>407728
Если ты про ловлю астероидов то там прирост небольшой.
Если про Тею то это совсем древние времена и тогда Пангеи не было.
Аноним 19/06/18 Втр 17:01:15  407742
DfwgDLDUYAAgB4U.jpg (418Кб, 1200x1197)
>>407734
Нет, там про то, что, мол, раньше, Луна была ближе, сутки длились меньше, месяцы были меньше, но их было больше, угол наклона Земли был нулевой и гравитация была слабее, поэтому, типа, динозавры и могли быть такими большими, не коллапсировать под собственным весом.
Аноним 19/06/18 Втр 17:20:32  407744
>>407742
Во времена динозавров сутки и правда были на час-два короче, но система Земля-Луна мало чем отличалась от современной, и земная гравитация была ровно такая же.

Динозавры и сейчас могли бы быть такими большими, и ноги бы под ними не сломались, это все брехня. Перенесенные в сегодняшний день динозавры бы, конечно, сдохли, но вовсе не из-за выросшей силы тяжести.
Аноним 19/06/18 Втр 21:50:30  407754
15176285164500.png (23Кб, 1106x458)
Я ньюфаг, поясните за плюсы такой вот вращающейся станции.
Аноним 19/06/18 Втр 22:17:46  407757
>>407754
Искусственная гравитация.
Аноним 19/06/18 Втр 22:41:07  407759
Супчика единственные кто делает прожоги ступени. Вопрос почему и как скоро это станет повсеместным?
Аноним 19/06/18 Втр 22:41:52  407760
>>407759
Фикс супчик на Спусикс
Аноним 19/06/18 Втр 23:04:49  407761
>>407754

Компактная станция с искусственной гравитацией и небольшой скоростью вращения при этом.
Аноним 20/06/18 Срд 15:04:57  407828
Вода уникальный элемент? Я слышал что она несжимаемая или что-то такое, поясните.
Аноним 20/06/18 Срд 15:09:18  407829
>>407828
Плотность жидкого состояния выше чем твердого, это все.
Аноним 20/06/18 Срд 15:10:20  407830
>>407829
Я не понял.
Да и разве не твердлое тело более плотное?
Аноним 20/06/18 Срд 15:40:27  407839
>>407829
а?
Аноним 20/06/18 Срд 15:44:05  407840
>>407830
У всех остальных - да, твердое плотнее жидкого. У воды нет. Но это не значит что ее нельзя сжать, в газовых гигантах есть разные формы льда с огромной температурой.
Аноним 20/06/18 Срд 16:00:03  407841
>>407757
>>407761
Спасибо :3

Аноним 20/06/18 Срд 16:59:33  407845
>>407840
>У всех остальных - да, твердое плотнее жидкого.

Кремний, галлий, германий, висмут, сурьма, церий и тысячи разных химических соединений с тобой не согласны, расширение при затвердевании довольно необычное, но нихуя не уникальное явление.
Аноним 20/06/18 Срд 22:20:41  407877
ч (6).jpg (60Кб, 500x500)
Меня озарило, и решил поискать гуглплее приложение, которое при наведении на реальное небо, все звезды подписывало. Я поставил то что сверху, но внутри оказалась только интерактивная карта, которая вроде как не работает в режиме виртуальной реальности. Мой реквест вообще осуществим?
Аноним 20/06/18 Срд 22:31:16  407878
>>407877
Heavens above norm rabotaet
Аноним 20/06/18 Срд 22:39:12  407879
>>407878
Там вроде только спутники.
Аноним 20/06/18 Срд 22:52:12  407880
>>407877
Star chart
Аноним 20/06/18 Срд 23:38:17  407881
>>407879
Планеты, если увеличить созвездия и звёзды.
Аноним 21/06/18 Чтв 01:15:49  407884
Как поведёт себя солнце, если всё его вещество перемешать до одинаковой плотности, температуры и состава?
Аноним 21/06/18 Чтв 01:22:55  407885
> Температура межгалактического газа составляет порядка десяти миллионов градусов.
это если туда положить предмет, он будет постепенно "разъедаться"? насколько быстро? или нас всех переживёт ?
Аноним 21/06/18 Чтв 05:10:13  407888
>>407884
И внезапно надобно уточнить до каких конкретно плотности, температуры и состава. Предположительно ты подразумеваешь параметры верхних слоев. Если до температуры в 6к К и плотности очень разреженного ионизированного газа, то обратный гравитационный коллапс очень большого газового шарика. А вот с составом уже сложнее.
Аноним 21/06/18 Чтв 07:33:43  407896
>>404988 (OP)
Меня мучал вопрос наша звездная система уже движется с какой то скоростью, но скрость света постоянна итд. Как мы можем прям уверенно говорить о том что вселенная расширяется? а не тут наблюдаем оптические иллюзии?
И в целом с какой скоростью движемся в млечном пути?
Аноним 21/06/18 Чтв 07:37:40  407898
>>407742
а Луна в конце концо улетит? или что
У меня есть только информация вроде, что если прокрутить путь вращение Луны то окажется что Луна никогда не была частью земли итп, но это знания полученные рандомно в школе.
Нигде немогу найти ответ на вопрос луна улетит с орбиты земли?
Аноним 21/06/18 Чтв 07:49:57  407901
>>407888
Нет, именно до средней. Например, адиабатически разделить солнце на кубометры и рандомно расположить их внутри солнечной сферы (на незанятых местах), повторить n раз. И охуевать с того, чё будет.
Аноним 21/06/18 Чтв 09:12:54  407903
Пацаны! У меня вопрос. Почему черные дыры охуительно сильно не светятся? Я думаю что это работает так: при подлете к черной дыре время замедляется, так? Чем ближе к горизонту событий, тем сильнее замедляется, верно?
В итоге из-за этого мы даже не можем увидеть как вещество звезды, которую разорвало притяжением ЧД упадет в черную дыру тк чем, ближе подлетает она к горизонту событий, тем сильнее время замедляется, тоесть там около горизонта должно висеть дохуища вещества, которое ОЧЕНЬ медленно падает в ЧД, так медленно, что мы даже не должны это видеть, верно? И получается, что там дохуя вещества скопилось еще очень-очень-очень старых звезд, верно? Почему тогда она не светится пиздец сильно?
Аноним 21/06/18 Чтв 09:21:00  407904
Безымянный.png (24Кб, 1243x648)
И вот еще вопрос. На каком расстоянии от черной дыры начинает замедлятся время? Из-за того что время замедляется свет до нас летил как бы медленнее, верно? Сейчас мы все время смотрим в прошлое, так ведь? Говорят мол звезда от нас в 10-ти миллионах световых лет от нас, значит мы видим ее такой какой она была 10 миллионов световых лет назад. Настоящего не существует вообще, лол.
Так вот и теперь представим что там рядом со звездой черная дыра. Я не знаю на каком расстоянии ее поставить, что бы она воздействовать начала. Так вот! Будет ли как-то это влиять на то какой мы ее будем видеть? Или все так же 10 миллионов лет назад?
Аноним 21/06/18 Чтв 09:40:34  407908
КосмонавтОлегАр[...].mp4 (1423Кб, 640x640, 00:00:24)
А можно ли спутничек летящий на МКС вручную оттолкнуть на другую траекторию? Выйти так в скафандре и ручками или ножками обнять, а потом выкинуть?
Аноним 21/06/18 Чтв 09:45:24  407909
>>407903
> время замедляется, так?
Это пиздёшь. Просто теоретический сладкий хлебушек в голове у математухов. Замедляется взаимодействие материи. Времени не существует как и расстояния, это всё абстракция, чтобы обезьянке было легче банан с пальмы доставать.
Аноним 21/06/18 Чтв 09:47:22  407910
>>407909
Ты опять выходишь на связь, ебанутый?
Аноним 21/06/18 Чтв 09:48:24  407911
>>407903
время замедляется же для вещества которое падает туда, а не для нас. но эффекты должны быть очень интересные при тщательном наблюдении.
Аноним 21/06/18 Чтв 09:49:53  407913
>>407903
Грубо говоря, вторая космическая у чд выше чем скорость света, то есть из нее ничего не может улететь. Чтобы мы могли видеть что-то, фотоны и прочие частицы должны улететь из дыры и долететь до наблюдателя, но им не хватает скорости.
Вещество там в основном от первоначальной звёзды и немного говна с округи.
>>407904
Радиус Шварцшильда. Для воздействия на звезду она должна быть очень близко, звёзда такого не выдержит и достаточно быстро "засосется" дырой.
Но если на безопасном расстоянии, возможно гравитационное линзирование если они встанут на линию, если такое найти можно пиздато разглядеть далёкие звезды.
Аноним 21/06/18 Чтв 09:52:27  407914
>>407908
с мкс ручками запускают
https://www.youtube.com/watch?v=lC0OOTg5XJ0
Аноним 21/06/18 Чтв 09:59:13  407915
>>407754
Намотает жи на камень. Или будут кружить с центром посередине. Я не ебу как там внатуре, но интуитивно так. Нужно видимо играться с массами и направлением тяги.
Аноним 21/06/18 Чтв 10:07:06  407916
>>407910
>Ты опять выходишь на связь
Да, а что? Поясни за время, что за сущность и как она отделима от планковской длинны и дискретно тактового пространства? Есть просто постоянное движение, а время это количество тактов смещения.
Аноним 21/06/18 Чтв 10:44:22  407917
>>407916
Сам придумал?
Аноним 21/06/18 Чтв 11:17:38  407919
>>407877
Star Walk 2.
Не уверен, что он actually фотографирует небо, а не выводит тебе полупрозрачную карту с привязкой к GPS и положению телефона, но вроде звёзды совпадали довольно неплохо, когда я по-пьяни с друзьями смотрел на небо в телескоп.
Аноним 21/06/18 Чтв 12:04:13  407923
>>407917
По аналогии с компьютером.
Аноним 21/06/18 Чтв 13:49:21  407929
>>407915
Чтобы намотало на камень, центр масс системы должен быть под поверхностью камня, а на картинке он снаружи.
Аноним 21/06/18 Чтв 14:40:08  407933
VKnpQoxxfxw.jpg (756Кб, 1638x2048)
Анон, помоги с мыслёй. В общем, представил себе фантастику и думаю над ней.
Значит ты, анон, летил на космическом корабле к другим звёзда, въёбся в тебя камушек и твою бандуру распидорасило и тебя выбросило на стабильную более-менее круговую орбиту вокруг неизвестной планеты в неизвестной тебе системе. Но в Штабе эту систему наверняка знают. У тебя есть вот-вот подохнущий передатчик, который выдержит только одну передачу. Тебе нужно масимально чётко и точно описать характеристики планеты, звезды и других планет системы, смотря из окошка/иллюминатора/кабины. И тут я подумал "А это вообще возможно?"
По сути, ты не знаешь ни размера планеты, ни массы планеты, ни расстояния до звезды - ничего. Ты только знаешь, за сколько времени пролетаешь от точки А до точки А. Допустим, ты летишь строго по экватору планеты.. И ты можешь засечь только время между пролётом утреннего экватора в первый и во второй раз, и это ещё при условии, что планета не вращается, потому что тогда и и сам терминатор сдвигается.
В общем, анон, ты обречён или как?
Аноним 21/06/18 Чтв 14:43:24  407934
>>407933
А источник сигнала твой штаб не может найти, да?
Аноним 21/06/18 Чтв 15:05:14  407936
15292505168900.jpg (868Кб, 1400x1009)
>>407934
Конечно не может. Это же мысленный эксперимент анона с двача, а не кого-нибудь там. А ты думал в сказку попал?
Аноним 21/06/18 Чтв 15:16:58  407937
>>407936
Больше похоже на мысленное дрочение вприсядку.
Без использования фантастических приборов для определения расстояния и параметров планет нужно наблюдение на протяжении многих орбит, а времени так понимаю не очень много.
Хотя примечательные системы типа трапписта можно на глазок узнать.
Или иные особенности, например планета с громадными кольцами и так далее.

Ещё есть вариант с ориентацией по окружающим звёздам.
Аноним 21/06/18 Чтв 15:44:17  407943
>>407933
Ты можешь примерно замерить по тому, как планета будет закрывать звезду
Аноним 21/06/18 Чтв 22:06:56  408018
>>407933
ШНН
Аноним 22/06/18 Птн 01:34:55  408043
149337034811341[...].jpg (84Кб, 600x895)
149338519915041[...].jpg (177Кб, 536x800)
Венера самплер. План такой:
Щвятой хэвик запускает межпланетный аппарат к венере, в составе аппарата буксир и ландер, буксил довозит добро до орбиты, ландер отделяется и мягко садится на поверхность используя плотность атмосферы, пока выкипает охладитель - бур делает из венеры не целку и забирает образец, образец помещается в возвращаемую ебу, еба надувает аэростат, отделяется от ландера и как пробка вылетает в верхние слои атмосферы, там доразгоняется при помощи небольшого двигателя. Буксир цепляет ебу на орбите и везет прямиком на мкс, где все заражаются спидораком.
Внимание вопрос, есть ли материалы для баллона аэростата что бы выдержать ад на венерианском дне и надуться? Как поведет себя газ при таком давлении и температуре, перейдет ли он в нужное агрегатное состояние, есть ли вообще такой газ? Если нет то может ли жесткая коробка наполненная газом обеспечить необходимый подъем что бы оказаться в нужных условиях и надуть уже нормальный шар? Дохуя ли дельты можно так выиграть, ведь венера меньше земли а атмосфера больше?
Аноним 22/06/18 Птн 02:41:43  408046
>>408043
Идея не нова, такие проекты были, но все упирается в то, что поверхность Венеры фактически нахуй никому не нужна.

https://www.jpl.nasa.gov/news/news.php?feature=5902

На Марсе хотя бы есть шансы высадить лысых макак, и даже вернуть их обратно, а Венера для людей 100% бесполезна, с тем же успехом можно высаживаться прямо в ад с чертями.

>там доразгоняется при помощи небольшого двигателя

Венера размером практически с Землю, твой «небольшой» двигатель должен выдать как минимум 7-8 км/с дельты, что автоматически означает минимальный вес ракеты в несколько тонн. Выполнимо, но нихуя не просто.

>Дохуя ли дельты можно так выиграть, ведь венера меньше земли а атмосфера больше?

Даже хуже, чем на Земле запускать с самолета, высота без скорости практически нихуя не дает.
Аноним 22/06/18 Птн 12:16:43  408091
Безымянный.png (10Кб, 499x556)
Предположим лежит себе анон на поляне и гоняет елду. Над ним пролетает спутник и останавливается до нулевой скорости прямо над аноном.
Как быстро упадет спутник на голову дрочиле? Или может он будет висеть в невесомости длительное время и анон просто свалит домой срать на двачах?
Пикрил примерная схема.
Аноним 22/06/18 Птн 12:19:36  408094
>>408091
Ускорение епта посчитай
Аноним 22/06/18 Птн 12:21:57  408095
>>408094
Я тупой, посчитайте за меня!
Аноним 22/06/18 Птн 12:24:09  408098
>>408091
Дрочила находится на крутящейся планете. Спутник промажет.
Аноним 22/06/18 Птн 12:24:48  408100
>>408095
Мне впадлу
Короче, зависит от высоты спутника
Аноним 22/06/18 Птн 15:41:56  408116
>>408046
>Даже хуже, чем на Земле запускать с самолета, высота без скорости практически нихуя не дает.
Чому хуже? Там же на поверхности давление под 90 атмосфер, почти как в камере сгорания, как там движок будет работать? И плотность выше в 60 раз. Земное давление аж на 50 км. Выглядит так, будто без шарика там как раз не обойтись.
Аноним 22/06/18 Птн 16:50:19  408123
15296590033230.png (20Кб, 1288x556)
>>408091
А если хочешь с учетом атмосферы - то это надо серьезно так обмазывать интегралами

Можешь просто глянуть для понимания
https://www.grc.nasa.gov/www/K-12/airplane/drageq.html
https://www.grc.nasa.gov/www/K-12/airplane/airsim.html
Аноним 22/06/18 Птн 21:02:13  408164
Аноны, Бетельгейзе может уничтожить человечество или нет?
Аноним 22/06/18 Птн 21:29:03  408170
>>408164
Нет, ты че ебанутый что ли?
Аноним 23/06/18 Суб 00:49:22  408204
>>407933

На глаз можно определить:
1. Класс светимости звезды (например Солнце это жёлтая звезда)
2. Цвет и общую характеристику планеты. (Например Земля это зелёно-голубая планета с атмосферой и её явлениями и большим спутником жёлтого цвета).
3. Расстояние до звезды в её диаметрах. (например Солнце на расстоянии 100 диаметров от Звезды)
4. Сделать вывод о соотношении радиуса и массы планеты по времени обращения. Время обращения легко засечь наблюдая положение светила.
5. Попытаться визуально найти другие знакомые звёзды. По угловым положениям нескольких звёзд найти будет ещё проще. (а знакомые звёзды на Млечном Пути)
6. Описать угол наклона экватора планеты относительно Млечного Пути, то есть общей плоскости эклиптики галактики.
7. Описать тепловое воздействие звезды на планету и твой корабль, по этому можно прикинуть и мощность звезды и вообще много чего ещё.
Аноним 23/06/18 Суб 00:57:15  408205
>>408043
>Дохуя ли дельты можно так выиграть, ведь венера меньше земли а атмосфера больше?

Дохуя.

>Как поведет себя газ при таком давлении и температуре, перейдет ли он в нужное агрегатное состояние, есть ли вообще такой газ?

Нормально, газ подобрать не проблема.

>Внимание вопрос, есть ли материалы для баллона аэростата что бы выдержать ад на венерианском дне и надуться?

ХЗ, я не знаю таких пластиков. Ни фторопластов, ни кременийорганических соединений. Это большая проблема.
Аноним 23/06/18 Суб 09:31:27  408234
>>408046
Тогда буксир сам превращается в аэростат и цепляет ебу в атмосфере а не на орбите.
возможно поднимать образец и не нужно, там ебанутая эррозия и все можно собрать из атмосферы
Аноним 23/06/18 Суб 14:35:06  408285
Как происходит выяснение обстоятельств катастроф КА? К примеру, когда какая-нибудь маленькая трубка в движке забивается говном, из-за чего ракета падает. Как узнают, что именно эта трубка именно что забилась именно этим говном? Разве аппарат не пидорасит?
Аноним 23/06/18 Суб 15:03:21  408288
>>408285
Ответ очень простой - в том числе и для этого там всё обмотано тыщами мильёнов самых разнообразных датчиков, передающих всё на землю в прямом эфире. Если, как в твоём примере, какая-нибудь трубка забилась, это поймут например по падению давления в той хуйне, которая получает <вещество-нейм> через эту трубку, и возможно по росту давления в той хуйне, которая в эту трубку <вещество-нейм> подаёт.
А главное, анализом аварий занимаются в том числе разработчики самой йобы - неглупые люди, которые очень хорошо знают, как работает эта техника, и примерно представляют, что могло привести к тому или иному результату (и так выяснить деталь-виновника аварии), а после этого могут представить, что могло привести эту самую деталь к отказу. Кроме телеметрии есть ещё множество видео запусков с разных камер (их реально очень много, намного больше чем видят рандомные непричастные хуи вроде нас с тобой, за стартами Шаттла камер 150 следило ЕМНИП), компьютерное моделирование и наземные испытания, в которых пытаются добиться повторения отказа.
В очень дедовые времена, когда самые дедовые деды учили Р7 и Атласы летать, такого компьютерно-телеметрического раздолья не было, и выяснять причины аварий было намного сложнее.
Аноним 23/06/18 Суб 15:35:47  408291
>>407898
из-за приливных сил
Аноним 23/06/18 Суб 16:36:35  408299
Есть ли открытые экзопланеты в кратных системах?
Аноним 23/06/18 Суб 16:53:51  408300
>>408299
Да, в том числе у ближайшей к нам звезды Альфы Центавра (тройной, а точнее 2+1 системе) есть экзопланета - Проксима Центавра b называется. Более того, она находится в зоне обитаемости своей звезды и, предположительно, землеподобная. Может даже с атмосферой. Даже рептилоиды, хоть и маловкроятны, но не исключены.
Аноним 23/06/18 Суб 17:02:14  408302
>>408300
Алсо, видисик про неё.
https://www.youtube.com/watch?v=zlP9g5nD5Lo
Аноним 23/06/18 Суб 17:02:58  408303
>>408302
*видОсик
Аноним 23/06/18 Суб 17:06:03  408304
>>408299
Да они вообще везде есть, даже в шаровых скоплениях. Только жизнь там не очень вероятна, ведь надо еще на синхронную орбиту встать, а также чтобы все звезды системы были стабильны.
Аноним 23/06/18 Суб 18:10:32  408313
>>408304
>везде есть, даже в шаровых скоплениях
х2. Даже у аккрецирующих тесных двойных, даже у пульсаров. В теории даже у чёрных дыр и даже в межпланетном (и даже межгалактическом) пространстве, без гравитационной привязке к какой-либо звезде. Даже вокруг Аллаха, поговаривают, десяток-другой шариков из говна вращается.
Аноним 23/06/18 Суб 18:18:22  408314
>>408313
фикс:
>в межпланетном
в межзвёздном пространстве, конечно же.
Аноним 23/06/18 Суб 20:44:12  408329
Чёрные дыры существуют, или всё таки нет? Если теория относительности верна, то должны же. Стивен Хокинг им всё таки почти целую жизнь посвятил
Аноним 23/06/18 Суб 20:49:15  408330
>>408329
Существуют, скоро даже фотографии дыры в центре галактики сделают.
Аноним 24/06/18 Вск 01:32:43  408350
>>408330
Фотографии чего, лол? Черного пятна? ЧД обнаруживают, когда вокруг чего-то невидимого вращаются куча звезд. Или если повезет - по диску аккреции
Аноним 24/06/18 Вск 01:38:29  408351
>>408350
Гравитационные волны же
Аноним 24/06/18 Вск 06:42:03  408376
>>408329
Существуют объекты, которые очень хорошо согласуются с теорией ЧД по косвенным признакам.

Невозможно доказать существование ЧД, так с ней нельзя взаимодействовать электромагнитно.
Ее нельзя трогать, облучать, наблюдать и т. д.

Все доказательства существования ЧД - косвенные (гравитация, аккреционные диски, джеты).

Однако их так много и они так хорошо укладываются в теорию, что существование ЧД давно считается доказанным.
Аноним 24/06/18 Вск 10:44:59  408386
>>408350
Ещё по возмущениям эфира можно обнаружить.
Аноним 24/06/18 Вск 10:55:53  408391
Как можно изменить скорости выше световой?
Аноним 24/06/18 Вск 11:02:51  408395
>>408391
Нельзя изменить то, чего не может быть
Аноним 24/06/18 Вск 11:03:23  408396
>>408395
Блять, меня тоже твой т9 заразил, измерить
Аноним 24/06/18 Вск 11:15:06  408398
>>408396
>>408395
Допустим, существуют некие частицы со скоростью выше световой. Как их измерить, если информацию о их движении мы будет получать со скоростью света?
Аноним 24/06/18 Вск 11:53:34  408399
>>408164
Если бы была в сто раз ближе к Земле то в момент бабаха могла бы подогреть атмосферу на несколько градусов, ну наверное могла бы и поднять радиационный фон на порядок другой, а так пофонит месяцок и угаснет.
Аноним 24/06/18 Вск 13:17:14  408401
>>408386
Слышал про эфир, вроде норм штука..
Аноним 24/06/18 Вск 13:52:16  408403
>>408401
Ато. Самая передовая теория.
Аноним 24/06/18 Вск 15:54:22  408412
>>408403
Да эт эфирный зомби наверное, ему не до теорий.
Аноним 24/06/18 Вск 17:08:30  408419
001.jpg (70Кб, 459x640)
>>404988 (OP)
Аноны, я правильно понимаю идею, что слепо дрочить на удельный импульс не совсем корректно? Ведь с ростом скорости истекающих газов проёбываемая энергия растёт быстрее тяги, и когда ракета ещё не разогналась - её полётный КПД будет низкий.

Т.е. получается, что выгоднее иметь на первой ступени короткоживущий РДТТ с низким удельным импульсом и ахуенной тягой. Второй ступенью обычный ЖРД с тягой поменьше и импульсом побольше. Третьей какой-нибудь ёба-литий-фторовый ЖРД, дающий маленькую, но очень скоростную струйку. Ну и на четвёртую ионник присобачить.

Короче, типа как на автомобильной КПП не стартовать с четвёртой, а повышать передачи по мере разгона.
Аноним 24/06/18 Вск 17:24:06  408420
>>408419
Примерно да, но по факту идеальный вариант это керосинка на первой ступени и водород на второй, двух ступеней достаточно для орбиты в большинстве случаев, а дальше по ситуации.
Для старта важна только общая тяга, низкий импульс не обязателен на связан при текущих технологиям желательно как можно быстрее набрать вертикальную скорость, потому что каждая секунда движения вверх отнимает 10 м/с дельты.
Например, именно из-за водорода на второй ступени Сатурн мог подняться намного больше Н-1, хотя первая ступень Н-1 попизже будет.
Тот же Сатурн имел не очень хорошую тягу первой ступени 1.2 твр вроде, поэтому гравитационные потери были ужасны.
Аноним 24/06/18 Вск 18:00:39  408422
>>408420
Ты еще забыл, что Н-1 - перетяжеленный кусок говна. С охулиардом ступеней.
Аноним 24/06/18 Вск 18:52:54  408424
>>408420
Например, я читал про какой-то упоротый проект паровой ракеты. Суть в том, что первая ступень помимо керосинки с окислителем имеет ещё бак с обычной водой. Вода подаётся прямо в сопло и там испаряется, отъедая часть тепловой энергии. Получается штраф на скорость реактивной струи (и на удельный импульс, соответственно), но прирост тяги за счёт массового расхода.
Аноним 24/06/18 Вск 19:23:56  408425
>>408424
Говно это и нахуй не нужно. У современных ракет проблемы только с импульсом, тяга заебись.
24/06/18 Вск 21:53:55  408434
>>408419
Посмотри на вторую пикчу в оп-посте. Помимо удельного импульса значение имеют конструктивное совершенство и ступенчатость. И обычно энергетическое и конструктивное совершенства в обратной зависимости. Для повышения массовой отдачи при прочих равных увеличение УИ полезно всегда, однако приходится искать наиболее выгодные соотношения по массам конструкций. А ещё для реальных изделий приходится брать во внимание стоимости и учитывать возможности разработки и производства.
Для совсем уж оптимизации размышлять не нужно, нужно решать соответствующие уравнения, учитывающие все факторы.
Аноним 24/06/18 Вск 22:12:12  408437
Почему нельзя запилить космодром где-нибудь в Сахаре и пускать оттуда атомолеты?
Аноним 24/06/18 Вск 22:13:12  408438
IMG2822.PNG (40Кб, 564x289)
Как они узнали, как выглядит наша собственная галактика?
Аноним 24/06/18 Вск 22:15:41  408440
>>408425
>Говно это и нахуй не нужно. У современных ракет проблемы только с импульсом, тяга заебись.
Тяги много не бывает. Больше тяги значит, что можно добавить к верхним ступеням больше топлива, заметно прибавив дельту.
Аноним 24/06/18 Вск 22:35:28  408443
>>408440
И импульс по пизде. И тогда первая ступень будет в разы тяжелее, а значит дороже.
Лучше поставить больше двигателей.
Аноним 25/06/18 Пнд 00:09:44  408459
Когда "Венеру" расконсервируют? Уже десятилетия без новых фоточек.
И какого хуя ещё на Меркурий не садились?
Ёбаный Марс, блядь, это всё он виноват своей "землеподобностью". То, что это пыльная радиоактивная пустыня имеет хоть какую-то сраную атмосферу, является привлекательным фактором в глазах ебучих нердов и некбердов со всего мира. НУ А ТО ЧОЩ, ВОН ПОЛЕТИМ НА МАРС, ЗАПАНУЭМО!! Тупые выродки, где мои охуенные фоточки Меркурия?!
Аноним 25/06/18 Пнд 00:25:15  408461
>>408459
На Меркурий через пару месяцев полетит аппарат, правда лететь будет 7 лет. Трудно туда сесть потому что встать на орбиту нужно дохуя дельты, плюс ещё посадка, короче нескоро.
На Венеру мутные планы, вроде как роскосмос хочет, но это значит ещё дохуя лет.
Аноним 25/06/18 Пнд 00:56:06  408463
>>408459
На Марсе лендеры живут годами, на Венере — единицы часов, куда бы ты послал аппарат, если бы была возможность? Ну и дело не только в количестве научных данных, но и в их качестве и полезности.

С полезностью сразу все ясно, поверхность Венеры для людей в ближайшие десятки лет абсолютно бесполезна, жить там нельзя, добывать нечего, вода давно улетучилась в космос, вывод чего-то на орбиту сложнее, чем на Земле. Даже солнечные панели работают хуево.

В научном плане тоже не шибко много интересного. Венерианская кора тонкая и относительно молодая (возрастом в несколько сотен миллионов лет), вся история планеты давно расплавилась и утонула в мантии. Тектоники плит нет, рельеф крайне однообразный — плоские лавовые равнины покрывают практически всю планету, изредка прерываясь возвышенностями и разломами.

В отличие от того же Марса, по камням которого миллиарды лет назад текли и высыхали реки, и в грунте остались следы глобального изменения климата, на Венере в историческое время не происходило ровным счетом нихуя.

Не понимаю этого дроча на Венеру, в общем, ну хуевая же планета, даже жалко просирать научные аппараты на подобную срань. Если что-то интересное там и есть, так только атмосфера, а сама поверхность максимально скучная хуйня.
Аноним 25/06/18 Пнд 01:02:54  408464
>>408463
Марс говно без будущего. На Венере при сайфайных технологиях можно нормально жить, на Марсе никогда, только в куполах, ну то есть ничем не лучше луны.
Аноним 25/06/18 Пнд 01:07:22  408465
1355943127688.jpg (33Кб, 300x300)
>>405170
У меня очень часто возникает вопрос. А почему очень много статей на научную тематику на вики, не переведено на русский? причём может быть на португальском, чешском или вьетнамском, но на русском очень часто не бывает. Это специально, чтобы школота/студентнота не могла нормально прочитать?
Вот только не надо щас начинать, что в 2018-м не знать инглиша и т.д. Есть гуглопереводчик, но сам факт отсутствия множества научных публикаций на вики на русском. Немного обидно даже как-то. Неужели всем похуй?
Аноним 25/06/18 Пнд 01:13:05  408466
>>408464
>На Венере при сайфайных технологиях можно нормально жить
Давление: 93 бар
Температура: 464 °C

Ваще изи там жить будет, курорт блять.
Аноним 25/06/18 Пнд 01:14:22  408467
>>408438
Ты когда-нибудь делал панорамную съёмку мобилкой?
Аноним 25/06/18 Пнд 01:15:53  408468
>>408464
При сайфайных технологиях на Марсе жить как раз попроще будет, чем на планете вообще без воды и с 90 лишними атмосферами газа, который некуда девать.

С Марсом рецепт прост, надо только нагреть немного и подбавить любого газа, и дальше все само пойдет, так как там все нужное для жизни есть прямо под поверхностью. Никакого правдоподобного способа убрать венерианскую атмосферу даже в фантастике еще не придумали.
Аноним 25/06/18 Пнд 01:19:01  408470
>>408459
Интерес к Венере был, когда никто не понимал, что там на поверхности.
Поэтому Союз и бомбил ее без остановки всякими "Венера №".
В 1975 году вроде бы оттуда фотки пришли (СУКА, В 1975 ГОДУ. ФОТКИ. С ВЕНЕРЫ. БЛЯДЬ, КРЕПКА БЫЛА СОВЕТСКАЯ ВЛАСТЬ)
Сейчас более менее понятно, в том числе, что там пизда и ничего, живущего больше часа туда не отправить. Поэтому экономически это нихуя не выгодно, максимум спутники ей сажать на орбиту (вроде там уже что-то узкоглазое крутится или должно вскоре).

А Марс - он и есть Марс, его относительно легко исследовать, плюс там точно была вода, что повышает интерес.
Аноним 25/06/18 Пнд 01:19:23  408471
>>408437
>Почему нельзя запилить космодром где-нибудь в Сахаре
Потому что все уже запилили там где им надо и удобно. кроме россии, у нас северный мордор, в любом случае проигрываем в полезной нагрузке.

>пускать оттуда атомолеты
Потому что атомолёты возможны только в твоём воспалённом мозгу.
Аноним 25/06/18 Пнд 01:20:07  408472
>>408468
Магнитное поле тоже обратно сможешь включить?
Аноним 25/06/18 Пнд 01:23:32  408473
>>408466
>>408468
Ну чё вы несёте?
Во первых Марс никогда не сможет удержать атмосферу. Там хуевая гравитация. Чтобы это изменить нужны такие высокие технологии, что когда они будут проще будет долететь до нормальных планет в других системах. А иначе это ничем не будет отличаться от поселений на луне, те же ебаные купола и закрытые помещения.
А концерты превращения Венеры в нормальное место придумали ещё в 70-80ых годах, например Саган предлагал много чего, причем без йоба технологий. Плюс вариант надувных объектов на высоте, там давление как на земле и лишь немногим жарче.
Аноним 25/06/18 Пнд 01:23:46  408474
>>408466
Генномодифицированные человеки там выживут.
Аноним 25/06/18 Пнд 01:24:54  408475
>>408471
> пук
Атомолеты возможны даже на технологиях 70-х, ебанашка.
Аноним 25/06/18 Пнд 01:25:24  408476
>>408472
Там оно есть по сути, особое взаимодействие с атмосферой даёт нормальную защиту. Плюс сама атмосфера блокирует значительную часть говна.
Аноним 25/06/18 Пнд 01:27:37  408477
Genius.png (121Кб, 811x624)
>>408330
>скоро даже фотографии дыры в центре галактики сделают
Фотографии того, что не выпускает за свои пределы фотоны.
Аноним 25/06/18 Пнд 01:28:45  408478
>>408476
Магнитное поле в 500 раз слабее земного.
Атмосферы почти нет. И нихуя она не блокирует.
Да и что она должна по-твоему блокировать и как?

Ты здоров, друг?
Аноним 25/06/18 Пнд 01:32:37  408479
>>408463
>>408470
Запускали же аэростаты "Вега" 1 и 2. И довольно долго работали они..
Аноним 25/06/18 Пнд 01:33:34  408480
>>408477
Передадут посредством хокингова излучения.
Аноним 25/06/18 Пнд 01:36:15  408481
>>408478
Бля я думал там про Венеру. У нее взаимодействие солнечного ветра с атмосферой создает аналог небольшого, но достаточно сильного, магнитного поля.
Аноним 25/06/18 Пнд 01:37:11  408482
>>408477
Ага, а сеть EHT это наверное мошенничество?
Аноним 25/06/18 Пнд 01:37:51  408483
>>408473
Это выдуманная проблема, потеря атмосферы это сотни миллионов лет, если б на Марсе была атмосфера как на Земле, то изменение плотности за несколько тысяч лет ты бы никаким прибором не зафиксировал.

>А концерты превращения Венеры в нормальное место придумали ещё в 70-80ых годах, например Саган предлагал много чего, причем без йоба технологий.

Ну-ка, перечисли их. Дай угадаю, начинаются эти твои маняконцерты, разумеется, со скинутых в атмосферу бактерий? Я тоже такое читал, только автор не знал, ни того, что бактериям для жизни тоже вода нужна в пропорциональных количествах, чего на Венере как-то не наблюдается, ни того, что связывание углекислого газа — обратимый процесс, и вывод из цикла жизни и захоронение углерода даже на Земле идет пиздец как медленно.
Аноним 25/06/18 Пнд 01:38:33  408484
>>408479
56 минут прожил спускаемый модуль.

Bitch, please
Аноним 25/06/18 Пнд 01:39:42  408485
b1e1f65e-a663-4[...].jpg (26Кб, 400x507)
>>408473
>Ну чё вы несёте?
Что ты несёшь, жирдяй-каникуляр. Это уже даже не троллинг тупостью. 464 градуса блять, какие к хуям человеки на венере? Даже не учитывая давление в 90атм и дожди из расплавленных металлов.

>Марс никогда не сможет удержать атмосферу
Она там уже есть, разряженная но есть, да и похуй на неё вообще, купола решат. Герметичный домик-гараж-ракушка с системой жсо и ты сможешь капчевать там сидеть, в защите от радиации песка и ветров. А прогуляться к васяну или еотовой, в соседнюю ракушку, сможешь напялив сайфайный скафандрик в обтяжечку, с подворотами. Или по подземному тоннельчику.
Аноним 25/06/18 Пнд 01:39:59  408486
>>408475
чо же ими не пользуются?
Аноним 25/06/18 Пнд 01:43:16  408487
>>408473
>Плюс вариант надувных объектов на высоте, там давление как на земле
Да, действительно, на высоте 25км над поверхностью венеры, давление 1 атм. температура около 25-35 градусов, правда ещё в нагрузку облака из серной кислоты, и радиация жёстче чем на марсе в несколько раз. Удачного освоения, толстячок.

покинул дебильный троллячий тред
Аноним 25/06/18 Пнд 01:48:28  408489
>>408483
А где ты атмосферу возьмёшь, для начала? Посчитай сколько массы нужно для комфортного давления прежде чем нести чушь про бомбёжку полюсов.
Кстати азот где брать будешь?

>>408485
Ты очком читаешь или как? В теории Венеру можно сделать пригодной для жизни в теории, Марс только в фантазии. Зачем жить на Марсе в куполах, если можно на луне?
Аноним 25/06/18 Пнд 01:48:28  408490
>>408484
> Каждый зонд проработал около 46 часов («Вега-1»: с 2:08 UT 11 июня по 00:38 UT 13 июня; «Вега-2»: с 2:07 UT 15 июня по 00:38 UT 17 июня; указано время получения сигналов на Земле).
Кто тут у нас ещё "bitch".
Аноним 25/06/18 Пнд 01:49:20  408491
>>408487
Пруфани радиацию или чепух.
Аноним 25/06/18 Пнд 01:49:56  408492
>>408489
Двачну, Марс - оверхайпнутая хуйня для фанатов "Звёздного пути" и прочих "manbaby"-задротов.
Венера - выбор чемпионов.
Аноним 25/06/18 Пнд 01:50:46  408493
>>408487
Какой же ты неженка. Небось ещё и на EM-Ведро надрачиваешь.
Аноним 25/06/18 Пнд 01:55:05  408495
>>408490
Ты просто читать не умеешь. 56 минут отработал посадочый модуль. А 46 часов атмосферный, который с посадочного взлетел на хуй знает сколько километров вверх, где совершенно другие давление и температура.
Аноним 25/06/18 Пнд 02:07:23  408500
>>408489
Она прямо в земле лежит и в полярных шапках, суммарно на Марсе достаточно одного только летучего углекислого газа примерно на 30-60% земной атмосферы, а еще воды дохуища. Азота также есть достаточно в почве, Кьюриосити его находил. На Марсе вообще дохуя всего во вмороженном или связанном виде есть.

Отмечу, кстати, что раз уж мы сравниваем с Венерой, то сразу надо задуматься, что проще, добавить 1 атмосферу газа или убрать 90? Добавить 20-30 градусов температуры или убрать 400+? Создать защиту от солнечного ветра вблизи солнца или в два раза дальше? Плюс никаких своих концертов ты так ни разу и не назвал, хотя уже несколько раз успел заявить, какие они охуенные и реалистичные.
Аноним 25/06/18 Пнд 02:11:51  408501
>>408500
Ты, главное, скажи, как это всё сделать.

А то пиздеть - не мешки ворочать.
Аноним 25/06/18 Пнд 02:13:06  408502
>>408501
Сначала ты про Венеру хоть на один пост распиши, а то мне надоело уже писать серьезно, а в ответ слушать троллинг тупостью.
Аноним 25/06/18 Пнд 02:14:56  408503
>>408500
Бля ты цифры нужной атмосферы посчитал или хотя бы поискал?
Количество азота какое? Как его выпускать?
Плюсы Венеры только в том что там есть нормальная гравитация и атмосфера, это все.
Аноним 25/06/18 Пнд 02:43:29  408504
>>408495
Я о нём и писал, ебан.
Аноним 25/06/18 Пнд 03:37:21  408506
NeptuneringsPIA[...].jpg (57Кб, 300x512)
Какие планеты и спутники могут быть потенциально хорошими кандидатами для создания человеческих колоний?
Аноним 25/06/18 Пнд 04:10:18  408507
>>408506
Это утопия, даже если бы они были, транспортировка к ним того количества грузов и людей, необходимых для существования человечества, даже в пределах СС для нынешнего физического принципа работы космических двигателей - невозможная задача.

И да - в СС таких мест нет. Через миллиард лет Солнце увеличит светимость до такой степени, что жизнь на первых четырех планетах будет невозможной в принципе.
Дальше только газовые гиганты и их крошечные спутники, ебимые приливными силами.

Смирись, ты умрешь, умрут все твои потомки и всё человечество. Затем Солнце спалит Землю, потом станет красным гигантом и окончательно сожжет, поглотив в себя Меркурий, Венеру и, возможно, Землю, а потом наступит один из предполагаемых концов Вселенной. И не будет ничего.
Поэтому live fast, die young.
Аноним 25/06/18 Пнд 06:28:37  408510
>>408438
>>408467
Добавлю про базовое черчение и построение фигуры по точкам.
Аноним 25/06/18 Пнд 06:41:31  408511
>>408507
ну в целом есть надежда на то что потомки разгадают больше законов физики чем мы и методв абузить её, еще лет 150 назад считалось что увсе в физике изучено и описано и мол мертвая наука.

Например как вариант наша вслеленная может не увеличиваться в размере а просто наблюдаемая часть заполнять пустое пространство во вселенной

Те фотоны не платят налоги за расширение вселенной и не тереяют энеригии. а может и правда все пизда
Аноним 25/06/18 Пнд 07:19:06  408513
>>408507
Ой, да вы заебали драматизировать.

Активное освоение СС будет после того, как люди наиграются с реюзом ракет и вкатятся в безракетный и частично безракетный запуск. Это тысячи тонн в сутки за копейки, даже с примитивными вакуумными трамваями в гималаях. Пара-тройка десятилетий при хорошем раскладе.

Транснептуновые и ближние межзвёздные перелёты будут после того, как завзеут ТЯРД. Ничего запредельного в нём нет, тупо постепенный прогресс сверхпроводников. 3Тл уже обыденность, 10Тл с трудом доступны. Ну а импульсный термояд на Z-машинах уже давно хуярит с положительным выходом с огромными запасом по температуре. Осталось сделать всю эту хуйню компактнее. Ожидаемый срок - начало 22 века.

Релятивистские корабли будут после того, как КПД производства антипротонов достигнет десятков процентов, а мировая энергетика сравняется хотя бы с солнечным светом, поглощаемым Меркурием. Опять же, ничего запредельного тут нет, тупо долгое и нудное масштабирование сегодняшных наработок. Срок 2-3 столетия.

Вот со сверхсветовыми всё уже сложнее, хотя зацепки есть. Пока мы их понимаем на пальцах, примерно как да Винчи понимл вертолёт. Вырастут энергопотребление, энергофокусировка и вычислительные способности порядков эдак на 8-9, тогда и поговорим о сроках.

Энивей, это не миллиарды лет нихуя. Миллиард лет можно восрать только если какое-нибудь политическое хуйло угробит на планетке высокоразвитую жизнь.
Аноним 25/06/18 Пнд 09:37:15  408520
>>408475
Заговор ракетной мафии.
Аноним 25/06/18 Пнд 10:28:18  408523
Ребята тупой вопрос, сколько стоит двигатель(установка) РД-8. Есть ли ссылка на цену?
Аноним 25/06/18 Пнд 10:33:23  408525
>>408465
Так получилось, что во второй половине XX в. английский язык стал чем-то вроде lingua academica ("язык науки", как в свое время латынь). Поэтому да, всем "похуй", т.к. азы языка выучить проще, а чтобы написать перевод более качественный, чем это умеет гуглопереводчик, нужно знать язык на достаточно высоком уровне, на что основная масса читателей научных статей (условных индусокитайцев) не имеют ни времени, ни желания.
Аноним 25/06/18 Пнд 10:51:19  408527
>>408513
Если Z-машины умеют выходить в плюс, почему итер является токамаком?
Аноним 25/06/18 Пнд 15:49:54  408550
>>408527
Потому что токамак более старая, проработанная и мейнстримная концепция, бросать которую на полпути не комильфо. Плюс он всё-таки более удобен - там просто ровно горит плазменный бублик, бери да снимай тепло.

Z-машина это такой экзотический симулятор процессов в водородной бомбе, который неожиданно выстрелил на 2млрд К. Тема эта новая, мутная, и конструктивно не очень удобная для электростанции - это надо гигантские конденсаторы быстро перезаряжать, снаряд менять с проволочками, и теплообменник туда каким-то хуем вкорячить.
А ещё распиаренный лазерный NIF поднасрал. Убили кучу бабла на эту йобу, а зажигания там так и не случилось. Что сделало чёрный пиар другим проектам импульсного термояда.

Поиск концепций при разработке чего-то принципиально нового это вообще цирк с конями. Можно в упор не замечать золотой самородок, можно выдрачивать изначально порочные вещи (и даже таки выдрочить до юзабельности), можно мерить интерес пропорционально въёбанным деньгам. Сейчас вот локхид-мартины вообще электростатический фузор из-под сукна достали и ебутся с ним.
26/06/18 Втр 00:03:02  408606
Зачем человеку космос если он такой слабак?
Писает, какает, не выдерживает перегрузок, радиации, мы же не сможем дальше марса улететь.
балтика9 26/06/18 Втр 01:58:19  408610
слыхал кто новость что взорвалась сверхновая и типа нам всем каюк? хотелось бы узнать будут ли губительные последствия
Аноним 26/06/18 Втр 03:24:54  408618
Без названия.png (125Кб, 1410x533)
Я не понял, так существование внеземной жизни это почти 100% факт? Т.е. не "ай вон ту билив", не "50 на 50", не идеи сумасшедших гиков и фантастов, а наша реальность?
Аноним 26/06/18 Втр 05:03:40  408620
>>408618
Внеземная органика - факт.
Добиологические химические реакции несложных веществ, включающие в себя самоорганизацию, конкуренцию и цикличность - факт.
https://ru.wikipedia.org/wiki/Реакция_Белоусова_—_Жаботинского
Богатые тяжёлыми элементами планеты с условиями, достаточными для существования жидких растворов - факт.
Самозарождение во всей этой хуйне какого-нибудь примитивного компилятора на РНК или аналогичном "языке биопрограммирования" - очень вероятное событие.
Самозарождение клетки, на 100% питающейся подножной неорганикой типа железобактерий - вероятное событие.
А вот дальше уже много вопросов. Ибо эволюция разгоняется долго, и на переход хотя бы к ссаным трилобитам нужно убить миллиард лет. И нет никаких гарантий, что за такое огромное время на планете не случится несовместимый с жизнью катаклизм.
С разумной жизнью вопросов ещё больше.
Аноним 26/06/18 Втр 10:44:49  408633
>>408618
Органика и органические молекулы это не значит жизнь в каком-либо виде. Это всего лишь те вещества, из которых состоит жизнь, но они вполне могут образовываться без жизни.
То есть всякая жизнь - органика, но не всякая органика жизнь.
Эта статья на картинке каким-то желтушным тоном написана. Например, на Земле метан является продуктом жизнедеятельности, а на Титане моря, реки и дожди из метана естественного происхождения.
Аноним 26/06/18 Втр 11:30:26  408635
1) Будет ли Как-Земля находится в приливном захвате с Как-Солнцем, если не будет какой-нибудь Как-Луны?
2) Разве способная планета промёрзнуть до ядра? Там же давления вообще адовые, жарко должно быть в любом случае
Аноним 26/06/18 Втр 11:38:03  408637
>>408635
Нет, нужно намного меньшее расстояние. У нас в системе даже маленький и одинокий Меркурий вплотную у Солнца копротивляется захвату.
Только если совсем крошечная, как астероид. Иначе давление создаст нагрев.
Аноним 26/06/18 Втр 12:38:25  408639
>>408620
>>408633
Спасибо
Аноним 26/06/18 Втр 12:40:35  408640
>>408635
Через 10 в нной степени лет любая планета промёрзнет до ядра, если не будет внешних аномалий.
Аноним 26/06/18 Втр 12:45:51  408641
>>408618
Представь себе что ты заходишь на сайт и там говорят, что ты выйграл миллион единственный на всю вселенную. Ты конечно можешь предположить что это так, но каковы шансы что это так? Ответом будет ответ на парадокс ферми.
Аноним 26/06/18 Втр 14:57:34  408663
>>408618
Ещё не 100%, вот когда джеймса уэбба запустят (постоянно откладывают, пидоры) - тогда и узнаем точно, есть ли фотосинтез на других планетах. Про обычные бактерии попытаться узнать можно, только продырявив другие планеты и спутники Солнечной системы.

Другое дело, что другая сложная жизнь может и не существовать в этой галактике. Смотря на свет от планет мы не поймём, что там: простые цианобактерии, медузы, стрекозы с два боевых истребителя размером или 10/10 няши лесные эльфиечки.

>>408635
1. Нет, даже наоборот - она бы быстрее вращалась. Солнце далеко, Земля тяжёлая, инерции у неё много.
2. Так-то должно, только за гугол лет в любом случае промёрзнет - закон сохранения массы-энергии никуда не делся.
Аноним 26/06/18 Втр 15:45:39  408667
>>408330
С уголком, три на четыре.
Аноним 26/06/18 Втр 16:21:52  408669
>>408610
Это в которой 200 световых лет уже другая галактика?
Аноним 26/06/18 Втр 16:33:27  408671
>>408610

Ну видишь как тебя сейчас жарит из космоса излучением радиации? Вот это и есть каюк.
Аноним 26/06/18 Втр 18:00:28  408672
Ребята, посоветуйте литературы про ядерные ракетные двигатели.
Аноним 26/06/18 Втр 20:42:55  408686
Сколько проживет условный посадочный аппарат на Ио?
Аноним 26/06/18 Втр 20:55:00  408688
>>404988 (OP)
Если поебаться в космосе - шлюха забеременеет? И каков будет плод?
Аноним 26/06/18 Втр 20:57:14  408690
>>408663
>Другое дело, что другая сложная жизнь может и не существовать в этой галактике
Да ну нахуй, земной ты шовинист. Хотя то, почему мы не видим другой очень высокоразвитой цивилизации - загадка. Неужели в космосе нет каких-нибудь пеленгов суперразвитых?
Аноним 26/06/18 Втр 21:01:42  408691
>>408690
Есть вариант что нам очень повезло и мы в числе первых.
Аноним 26/06/18 Втр 21:38:52  408696
>>408506

Марс, крупные спутники Сатурна и Юпитера.
Для начала необходимо научиться сохранять биосферу Земли и вообще бережно к ней относиться. Потому что лет через 50 проблемы с экологией могут стать основными.
26/06/18 Втр 22:20:46  408705
p0002.jpg (48Кб, 600x863)
>>408672
Аноним 27/06/18 Срд 03:13:30  408731
школьник 7.jpg (151Кб, 491x500)
>>408507
>Это утопия,
Ш, ты опять выходишь на связь?
Аноним 27/06/18 Срд 03:39:43  408734
>>408731
Тебя то кто спрашивал, псина?

И да - где перекот?
Аноним 27/06/18 Срд 07:18:42  408739
>>408468
> Никакого правдоподобного способа убрать венерианскую атмосферу даже в фантастике еще не придумали
Пол Андерсон, "Большой дождь"
Аноним 27/06/18 Срд 10:57:13  408745
>>408734
>перекот
Тред еще с нулевой не смыло, рачок с Энцелада.
Аноним 27/06/18 Срд 12:45:11  408750
>>408739
>Пол Андерсон, "Большой дождь

Фантастика, нисколько не правдоподобная, и даже сомнительно научная. Ради интереса открыл, уже в самом начале появляются такие правдоподобные вещи, как натуральные демоны Максвелла.

>Трубы захватывали несущийся воздух и разделяли быстрые и медленные молекулы; более холодная фракция направлялась в систему охлаждения, поддерживающую в городе окружающую среду, обеспечивающую нормальную жизнедеятельность людей — снаружи температура превышала точку кипения воды.

Венера, опять же, не имеет ничего общего с настоящей. Основная идея рассказа — это то, что на Венере дохуища воды, просто в связанном виде, и люди пытаются ее выпустить (тот самый большой дождь). Ничего подобного, разумеется, в реальности нет: всю воду давно разбило на атомы солнечным ветром и водород сдуло в космос, в атмосфере Венеры и поверхностном грунте вода практически полностью отсутствует и сделать ее не из чего. Этот факт также автоматически отменяет бактерии, которыми в рассказе производится кислород из углекислого газа.

Про воздушные насосы и говорить смешно, ими атмосферу не переработать не то что за двадцать лет, а за двадцать тысяч бы успеть. Атмосфера, опять же, далека от настоящей, в рассказе она в основном состоит из формальдегида и других углеводородов и гораздо менее плотная, так что задача колонистов состоит больше в изменении состава атмосферы, чем в ее практически полном удалении. Никакого способа захоронить сотни квадриллионов тонн газа даже не упоминается.

Ну и плюс халявная энергия у них там повсюду — много месторождений ядерного топлива, солнце ярко светит до самой поверхности, везде дуют постоянные и сильные ветры. Ни того, ни другого, ни третьего на поверхности настоящей Венеры нет.

В общем, хуйня какая-то, для 50-х годов было норм, но после того, как АМС выяснили настоящее положение дел, читается больше как юмористическое фентези.
Аноним 27/06/18 Срд 14:45:12  408762
>>408690
В 50 световых годах точно нет (иначе бы уже уловили и ответили), да и дальше вряд ли - ни сфер дайсона, ни искусственных черных дыр нет.

Антропный принцип. Так вышло, что в этой галактике/части галактики люди - первый разумный вид, если бы им были пеленги - уже они бы думали, почему сверхразвитых людей нет.

Даже если жизнь не редкая, ей надо 1) дохуя времени (спокойная звезда, за все время существования жизни остающаяся в спокойном районе, и спокойная планета, впрочем не слишком.) 2) дохуя энергии (жить на поверхности) 3) удача (появление митохондрий - очень маловероятная вещь, и вообще хуй знает как это случилось. В меньшей степени это применимо к многоклеточности, вторичноротости, разуму)
Аноним 27/06/18 Срд 16:49:36  408772
>>408762
А как ты отличишь искусственную черную дыру от природной, а, мегаинженер?
Аноним 27/06/18 Срд 17:13:42  408773
Почему Бразилия гораздо ближе к 0° меридиану, он даже проход по Бразилии, но считается что "жарко" в Нигерии, которая от него значительно севернее? Почему в Бразилии нету пустыней? Почему бразильцы не чернокожие?

Правильно ли я понимаю, что чем ближе к 0° меридиану, тем менее отчетлива смена сезонов года, а на экваторе её вообще нету? Т.к. лето, осень, весна и зима здесь почти что одинаковые? Или за счет смены сезонов в других областях формируются всякие муссоны и циклоны, которые идут к экватору и портят праздник?
Аноним 27/06/18 Срд 17:14:26  408774
>>408773
>Т.к.
т.е /fix
Аноним 27/06/18 Срд 17:20:15  408775
Я еще Нигер с Нигерией перепутал, но не суть, они соседи. Нигер, аазазазаз, почему они еще не поменяли название и не обиделись?
Аноним 27/06/18 Срд 17:45:02  408776
>>408773
В Бразилии хорошо с дождями, много (относительно Нигера) пасмурных дней, значительно меньше часов солнечной активности в день (и соответственно ниже температура) и много обеспечивающей тень растительности, которая гарантировала, что безволосые обезьяны не сгорят дочерна.
В Нигере с осадками всё очень плохо, часов солнца очень много, растительности нет, облаков нет, тени нет, обезьяны торчат на солнце 24/7.

Осадки как-то связаны с физической высотой страны (бразилия едва возвышается над морем, нигер высокий) и удалённостью от тропиков (сходящиеся к экватору воздушные массы забирают воду оттуда и сбрасывают на экваторе).
Аноним 27/06/18 Срд 18:25:56  408778
1086px-EarthGlo[...].png (414Кб, 1086x858)
>>408773
Общая циркуляция атмосферы такая, в Бразилию в основном дует ветер с океана, а в северную Африку с континента. На западной стороне Южной Америки, например, на той же широте пустынь дохуя, потому что вся влажность успела осесть в Андах.

Бразильцы в основном метисы, микс из португальцев и индейцев, приправленные африканскими неграми, и выглядят соответствующе, от практически 100% европейцев до натуральных нигр.
Аноним 27/06/18 Срд 19:34:59  408780
>>408778
Что за потоки обозначаемые маленькими красными овалами расположенные выше больших красных? Есть что про них интерестного известно?
Аноним 27/06/18 Срд 19:41:55  408781
>>408772
В искусственную срут чтобы компенсировать излучение Хокинга.
Аноним 27/06/18 Срд 19:52:17  408784
>>408781
Ох ебать там излучение мощное, за сколько тысяч лет дыра солнечной массы килограмм высрет?
Аноним 27/06/18 Срд 20:24:41  408787
>>408784
А если маленькая дыра и не кончается - значит срут.
Как Солнце смысла нет делать, там выхлоп слабый будет.
Аноним 27/06/18 Срд 20:41:09  408789
Навеяно >>408783

А сколько уже денег вложили в него? Насколько реально отказаться от JWST и перенаправить деньги на другие проекты? Или сократить научную программу, выкинуть 2/3 неработающего, проблемного и смежного оборудования, поработать напильником и запустить как есть?
Аноним 27/06/18 Срд 20:51:09  408790
>>408789
Вложили 8,8 миллиардов, еще миллиард в процессе. Реальность отказа — 0,1%, реальность запуска как есть — 0%.

Уэбб уже построен и безумные миллиарды уже потрачены, вернуть их нельзя. Запустить как есть означает немалую вероятность того, что работать он не будет вообще, а отремонтировать на орбите, как Хаббл, его не выйдет — слишком далеко.

Лишнего оборудования там почти нет, это тебе не исследователи каменюк с миллиардом разных инструментов и задач, а просто инфракрасный телескоп, только очень большой.
Аноним 27/06/18 Срд 21:34:05  408793
>>408790
Ещё миллиард от Европы.
Аноним 27/06/18 Срд 22:39:25  408803
Почему Оу-мяу-мяу считается первым обнаруженным межзвёздным объектом, в то время как ещё в 2015 установили, что звезда Шольца пролетала через Солнечную Систему всего несколько десятков тысяч лет назад? (doi:10.1088/2041-8205/800/1/L17)
Аноним 27/06/18 Срд 22:42:08  408804
>>408803
Первым наблюдаемым. Существование подобных залетный камешков никогда не ставилось под сомнение.
Недавно кстати ещё одну хуйню нашли на ретроградной орбите солнца, тоже залетыш скорее всего.
Аноним 28/06/18 Чтв 00:29:15  408813
>>407904
> И вот еще вопрос. На каком расстоянии от черной дыры начинает замедлятся время?
На любом блять.
Но с разной степенью.
> свет до нас летил как бы медленнее, верно?
Нет, скорость света постоянна
Аноним 28/06/18 Чтв 01:59:27  408818
>>408813
>На любом блять.
>Но с разной степенью.
Но если подумать, гравитационные волны распространяются тоже со скоростью света. То есть, схлопнулись говна в точку - и мы узнаём что это чёрная дура только через N десятков, сотен, миллионов лет.

А если ещё сильно подумать, из-за малой скорости света Вселенная вообще обладает очень хуёвой причинно-следственной связью своих кусков друг с другом??? у меня мозг лопается от разрыва шаблона
Аноним 28/06/18 Чтв 02:03:55  408820
>>408818
Если подумать вообще заебательски - куски процессора, который обрабатывает высранное на дваче говно, тоже имеют очень хуёвую связность, т.к. слишком медленная скорость света играет заметную роль сравнительно с размерами кристалла и частотой. Так что ничего удивительного.
Аноним 28/06/18 Чтв 02:37:00  408826
>>408818
Не совсем. Например у нас уже нет причинной связи с объектами на краю наблюдаемой вселенной, но но у объектов на середине пути есть связь с объектами за границей вселенной. И так далее.
Аноним 28/06/18 Чтв 03:31:10  408827
Посоветуйте хороший учебник по дифференциальному и интегральному исчислению, надо вспомнить школьную базу и восполнить пробелы в образовании. Хорошо бы и теорию, и сборник задач с ответами и, желательно, кратким решением. Не сильно мудреную, мне не экзамены сдавать, а для общего развития + понимание предмета задачи и пути её решения + освоить практику вычислений с калькулятором.

Какой-нибудь мат.анализ для студентов с непрофильным по математике техн.образованием? Или мб курс математики для головастых 11-х классов?
Аноним 28/06/18 Чтв 07:59:36  408830
image.png (1099Кб, 900x600)
Мне нужно проконсультироваться по поводу одной штуки. Если у планеты есть кольца, то они будут видны с поверхности ночью или получится всратая тень от планеты? Или всё зависит от разных факторов? Луна так-то вроде как отражает солнечный свет, хотя с другой стороны, есть же и лунные там затмения всякие. Хз бля...
Аноним 28/06/18 Чтв 08:32:48  408831
15129846333430.png (482Кб, 1366x768)
>>408830
Открой спейсдвижок, да погляди, пикрелейтед.
Аноним 28/06/18 Чтв 11:12:48  408845
>>408827
Зельдович "Высшая математика для начинающих" — лютая годнота.
Аноним 28/06/18 Чтв 13:29:36  408863
>>408845
Спасибо, гляну
Аноним 28/06/18 Чтв 14:41:55  408867
Давайте перекат уже, заебало каждый раз вниз страницу мотать.
Аноним 29/06/18 Птн 07:13:54  408961
ARYb-k6q0vU.jpg (155Кб, 1280x856)
Анон, пикрилейтед лжёт? Может ли такое количество водорода и гелия, даже разогретого в ядре до 15млн. градусов выжечь кислород и убить всё живое?
Я пробовал гуглить, но кроме ссылок на научпоп и вялых сомнений в комментариях ничего не нашёл.
Аноним 29/06/18 Птн 07:41:12  408962
https://youtu.be/J0ldO87Pprc
Аноним 29/06/18 Птн 07:41:43  408963
>>408962
–>
>>408961
Аноним 29/06/18 Птн 07:41:46  408964
>>408961
Ну явно подразумевается термоядерный взрыв. Расчеты надо проверять. Само по себе понятие "булавочная головка" слишком расплывачатое. Однако причем тут поглощение кислорода - на этом мои полномочия всё.
Аноним 29/06/18 Птн 08:39:50  408965
>>408962
Благодарю
Аноним 29/06/18 Птн 12:13:22  408996
>>408961
Какое в-во имеется ввиду?
Средняя плотность Солнца меньше плотности воды, так-то. Поправьте меня.
Аноним 29/06/18 Птн 15:19:15  409029
>>408996
Средняя плотность таки повыше будет (на 40% больше плотности воды), но даже из самого ядра если 1 кубический сантиметр взять, нихуя не произойдет.

Это всего лишь 150 грамм водорода, пусть даже и нагретого до 15 млн градусов, и имеющего внутреннюю энергию порядка 32 гигаджоулей (~7 тонн тротила). Термоядерного взрыва никакого, разумеется, не будет (в солнечном ядре даже при ебическом давлении и так реакции очень медленно идут, тепловыделение в пересчете на килограмм там ничтожное), химической энергией тоже можно пренебречь.

На расстоянии 160 км даже не услышат ничего.
Аноним 29/06/18 Птн 16:08:46  409033
>>409029
>160км и не услышат
Ну и похуй на эту порашу. Семь килотонн хорошо потрудятся на пользу человека.
Аноним 29/06/18 Птн 16:10:06  409034
>>409033
Ну тонн же. Мало, пиздц. Надо больше.
Аноним 29/06/18 Птн 16:51:09  409041
А самые большие звёзды типа чёто-там большого пса лень гуглить, у них вообще внешняя часть гравитационно связана с ядром? А то ведь так и планетарную туманность можно звездой назвать и продолжать охуевать от размеров.
Аноним 29/06/18 Птн 17:12:30  409042
>>409041
Может быть связана, а может, и нет.

Радиус звезд обычно меряют по оптической толщине, т.е. поверхность, где фотоны перестают сталкиваться с веществом звезды и начинают свободно вылетать в космос (более точно — менее N столкновений в среднем, обычно N=1) и считается поверхностью звезды, а дальше уже корона.
Аноним 29/06/18 Птн 17:15:44  409043
Если до завтра не перекатите, я сам перекачу, и могу сразу сказать, что вам не понравится, как я это сделаю.
Аноним 29/06/18 Птн 17:22:04  409044
>>409042
Ну тогда хууууй знает, сверхновые должны быть самыми большими, там наверно до световых недель-месяцев такие условия сохраняются. А по-пацански надо только на главной последовательности мерить, когда со звездой нет никаких неожиданностей и есть уверенность в завтрашнем дне
Аноним 29/06/18 Птн 17:43:10  409045
201[1].jpg (648Кб, 990x767)
>>409044
С чего бы это? Сброшенная оболочка не такая плотная, рассеивается на ней совсем немного света (примерно я так почувствовал), явно не каждый второй фотон.

А по-пацански это когда у звезд вообще перестает существовать радиус? Как насчет пульсирующих звезд, где поверхность сжимается и расширяется, у них тоже радиуса нет? Хуйня какая-то, в общем, измерение по оптической толщине пусть и не идеально, зато работает на всех звездах и позволяет сравнивать их между собой.
Перекат Аноним 30/06/18 Суб 14:21:41  409139
GettyImages5414[...].jpg (62Кб, 800x781)
>>409138 (OP)
>>409138 (OP)
>>409138 (OP)
>>409138 (OP)
>>409138 (OP)
>>409138 (OP)
Аноним 01/07/18 Вск 02:50:29  409174
>>408669
да хуй пойми, везде разное пишут
то 200 св. лет, то 200млн , то вообще они уже не в курсах на каком расстоянии
Аноним 01/07/18 Вск 07:32:17  409191
>>408775
Обижаются совсем другие люди, которые даже в египте не были, не говоря уж о остальной африке. Для них же это нормальное название.
Аноним 21/08/18 Втр 19:06:06  417512
>>408688
Такой же будет, вопрос только в развитии в условиях невесомости.


Топ тредов
Избранное